You are on page 1of 40

CMA 1 D

Domestic Insitutional
Environment of Business
128 Questions

[5] Source: CMA 0687 1-13


Antitrust laws
A. Set maximum allowable prices for firms in
oligopolistic industries.
B. Prohibit firms from collecting price information.

[1] Source: CMA 0685 1-31


Sunset laws refer to
A. Government support for major business firms in a
state of decline.

C. Require firms with high earnings to provide price


and quantity data to the Federal Trade Commission.
D. Prohibit firms from allocating customers.

B. Protection of employees nearing retirement.


C. Regulatory statutes applicable to certain defense
contractors.
D. A periodic review of and a fixed termination date
for government programs.

[6] Source: CMA 0687 1-15


Tie-in sales (e.g., the sale of camera and film together) are
legal
A. If the tie-in is necessary to assure product quality.
B. If the tying product is patented.

[2] Source: CMA 1286 1-23


Currently, corporations are chartered by

C. If used to facilitate price discrimination.


D. Under all circumstances.

A. The federal government.


B. State governments.

[7] Source: CMA 0687 1-22


The Federal Communications Commission

C. The Securities and Exchange Commission.


A. Allows free access to the radio spectrum.
D. The board of directors of the respective
corporations.
[3] Source: CMA 1286 1-28
Which one of the following statements is a correct
characterization of current federal policy toward political
action committees (PACs)?
A. Corporations may use company funds to organize
and administer a PAC.

B. Limits the number of television stations the major


television networks may own.
C. Regulates the profits of radio and television
stations.
D. Censors program content on television.
[8] Source: CMA 1287 1-14
An example of social regulation of business is

B. Corporations may make corporate contributions


to PACs.

A. Tariffs on imported lumber products.

C. Unions may help organize PACs, but corporations


may not.

B. Rules against discrimination in the hiring of


employees.

D. Individuals may only make political contributions


through PACs.

C. Enforcement of patent laws.


D. Antitrust rules agreements to allocate customers.

[4] Source: CMA 1286 1-26


The recent legal concept of strict liability in tort
A. Requires consumers to prove that producers were
negligent.
B. Requires the proof of breach of warranty.
C. Permits contributory negligence on the part of
consumers to be an acceptable defense by
manufacturers.

[9] Source: CMA 1287 1-16


Requirements by licensors and franchisors that their
licensees and franchisees buy inputs from a particular
supplier
A. Are legal if they are necessary to assure product
quality.
B. Are always illegal unless the input is patented.
C. Are legal if used to facilitate price discrimination.

D. Tends to support plaintiff claims against


manufacturers of products that are judged to be
inherently dangerous.

D. Are legal under all circumstances.

[10] Source: CMA 1287 1-17


The intent of antitrust laws is to
A. Establish a range of allowable profit rates for firms
in oligopolistic industries.
B. Prohibit firms from engaging in joint ventures with
foreign firms.
C. Require firms with high earnings to relinquish any
exclusive patent rights which they own.

If Federal Aviation Administration regulation of airline


passenger service increases the variable costs of individual
firms in the industry by requiring greater expenditures on
quality checks and maintenance, then
A. In the long run, the industry will tend to have more
firms producing lower levels of output.
B. Fixed costs will rise.
C. Prices will fall in the long run.
D. Prices will rise in the short and long run.

D. Prohibit agreements that limit individual firm


output.
[11] Source: CMA 1287 1-18
Which one of the following statements regarding the
application of antitrust rules to professional fees is true?

[15] Source: CMA 1289 1-26


The two major functions of the Federal Trade Commission
are
A. Antitrust actions and consumer protection.

A. Most professionals, including physicians, lawyers,


engineers, and accountants, are exempt from the
antitrust laws.

B. Antitrust actions and regulation of import quotas.

B. Agreements to increase fees are legal if the quality


of services provided improves.

D. Consumer protection and monitoring labor union


practices.

C. Fee discrimination in any form is illegal.


D. Agreements to set either minimum or maximum
fees are illegal.

C. Regulation of railroads and airlines.

[16] Source: CMA 1289 1-24


All of the following are characteristic of the Taft-Hartley
Act except
A. The prohibition of a closed shop.

[12] Source: CMA 1287 1-19


The antitrust prohibitions against mergers that would tend
to lessen competition
A. Are enforced solely by the Commerce
Department.
B. Are enforced solely by the Federal Trade
Commission.
C. Are enforced by the Federal Trade Commission
and the Justice Department.
D. Were repealed by an Act of Congress in 1982.

B. The guarantee of the workers' right to organize.


C. States are encouraged to pass right-to-work laws.
D. The provision of descriptions of unfair labor
practices committed by unions.
[17] Source: CMA 1289 1-29
The basic purpose of the securities laws of the United
States is to regulate the issue of investment securities by
A. Providing a regulatory framework in those states
that do not have their own securities laws.
B. Requiring disclosure of all relevant facts so that

[13] Source: CMA 1287 1-20


In regulating intrastate local utilities, state public utility
commissions
A. Usually approve rate schedules that set the same
rates for residential, commercial, and industrial
customers.
B. Use only original cost methods of rate base
valuation.
C. Permit utilities to refuse service to some customers
within their jurisdiction.
D. Usually set rates of return for utilities based on the
cost of capital as reflected in the capital markets.
[14] Source: CMA 1287 1-21

investors can make informed decisions.


C. Prohibiting the issuance of securities that the
Securities and Exchange Commission determines are
not of investment grade.
D. Channeling investment funds into uses which are
economically most important.
[18] Source: CMA 1289 1-30
Which one of the following statements best describes how
regulatory agencies of the U.S. government are restricted in
the adoption of specific regulations?
A. Regulations must be consistent with standards
established in the congressional act which created the
agency.

Commission (EEOC) is
B. All proposed regulations must be cleared with a
central coordinating agency, the General Accounting
Office.
C. The agencies must present proposed regulations
to all affected parties for comment.
D. Businesses subject to the regulation must be
notified 1 year before the regulation will be put into
effect.
[19] Source: CMA 0690 1-2
The Consumer Product Safety Commission (CPSC) has
been called by its critics "the most powerful regulatory
agency in Washington." The most likely reason for this
concern is that the CPSC
A. Covers all consumer products.
B. May ban the production and sale of a product until
it has formulated a standard.

A. To restrict enforcement of the Equal Employment


Opportunity Act of 1972 to relatively small
companies in order to minimize disruptions.
B. To rank the goal of employee productivity ahead
of the goal of equal employment opportunity
considerations.
C. To avoid legal remedies to achieve the goals of the
EEOC.
D. To have businesses achieve employment mixes
reflecting the local labor pool of protected groups.
[23] Source: CMA 0691 1-24
State laws that make it illegal to require union membership
as a condition for getting or holding a job are known as
A. Fair trade laws.
B. Fair employment laws.

C. Does not permit an industry group to adopt a


voluntary standard.
D. May order the recall of products it has determined
to be unsafe.
[20] Source: CMA 0690 1-5
Truth-in-lending is one form of price standardization that,
since the adoption of the Consumer Credit Protection Act
on July 1, 1969, has been provided by U.S. government
regulations. The purpose of this legislation is to
A. Regulate the amount of interest that may be
charged.

C. Open shop laws.


D. Right-to-work laws.
[24] Source: CMA 0691 1-30
An industry that has not experienced significant
deregulation is the
A. Airline industry.
B. Banking industry.
C. Trucking industry.

B. Prohibit the use of appraisal fees.


D. Steel industry.
C. Allow immediate wage garnishment by creditors.
D. Disclose the finance charge and the annual
percentage rate.
[21] Source: CMA 0690 1-6
Equal employment opportunity is concerned with, among
other issues, the recruitment and selection of employees. A
U.S. Supreme Court ruling in 1971 involved the use of
pre-employment tests (Griggs v. Duke Power Co.). In that
case, which was determined under Title VII of the Civil
Rights Act of 1964, the Court ruled that pre-employment
tests
A. Given to minorities, regardless of the reason given,
violate Title VII.
B. Are legal, as long as there is no intent to
discriminate.
C. In any form are illegal.
D. Must be directly related to job requirements.
[22] Source: CMA 0691 1-21
A major policy of the Equal Employment Opportunity

[25] Source: CMA 1291 1-19


Over the years, various groups and activities have been
exempted from coverage by the U.S. antitrust laws. Which
one of the following statements regarding such exemptions
is correct?
A. All sales and purchases of products by American
firms outside of the U.S. are automatically exempt
from coverage by the antitrust laws.
B. All resale price maintenance agreements between
manufacturers and retailers selling the manufacturers'
product brands are exempt from the federal antitrust
laws.
C. Under certain conditions, a person who wishes to
form an export trading company may obtain a
certificate of antitrust immunity from the Commerce
Department after concurrence by the Justice
Department.
D. Water common carriers in foreign trade may enter
into price fixing and market sharing treaties or
agreements provided the agreements are ratified by
the U.S. Senate before being put into effect.

[26] Source: CMA 1291 1-20


The Environmental Protection Agency (EPA) might control
pollution by setting effluent standards for maximum
discharge or by constructing a sliding-tax charge based on
the amount of effluent emitted. One advantage of the
sliding-tax charge is that

The general approach to regulation by the Consumer


Product Safety Commission is to
A. Levy fines against the producers of unsafe
products.
B. Set safety standards for various products.

A. It will totally eliminate pollution.

C. Increase the freedom of consumer choice.

B. There is greater business incentive to discover new


methods of controlling pollution.

D. Reduce the cost of regulated products.

C. It increases the necessity for the EPA to fully


understand pollution control technology.
D. It increases the degree of direct government
intervention in the economy.

[31] Source: CMA 1291 1-26


Social regulation, as exemplified by the Occupational
Safety and Health Act and by the Environmental Protection
Act, is frequently alleged to be inefficient. The perception
by firms is that this inefficiency appears to be a result of
A. Concern for the quality of life.

[27] Source: CMA 1291 1-21


With respect to the federal antitrust laws, regulated
industries are
A. Completely exempt.

B. Lack of concern for the magnitude of marginal


benefits relative to marginal costs.
C. The use of flexible rather than rigid standards by
bureaucrats.

B. Covered to the same extent as private industries.


D. Low compliance costs to business firms.
C. Covered to the extent determined by the
applicable regulatory agency.
D. Covered to the extent determined by statute and
the courts.
[28] Source: CMA 1291 1-22
Federal regulatory agencies do not have power to
A. Impose agency taxes on private industry.
B. Issue rules and regulations.

[32] Source: CMA 1291 1-28


The National Labor Relations Act, also known as the
Wagner Act, guarantees labor certain rights. This Act
A. Requires management to make detailed financial
reports to the National Labor Relations Board.
B. Designates a list of unfair labor practices on the
part of unions.
C. Guarantees the right of self-organization and the
right to collective bargaining with management.

C. Investigate violations of statutes and rules.


D. Recommend penalties for violations of statutes
and rules.
[29] Source: CMA 1291 1-24
Violations of federal regulations on equal employment
opportunity are sometimes inferred from the fact that a firm
has a very small percentage of protected groups among its
employees. These regulations are sometimes attacked on
the grounds that they may be in conflict with the economic
concept that

D. Requires that elections of union officers be


regularly scheduled and that secret ballots be used.
[33] Source: CMA 1291 1-29
The Taft-Hartley Act attempted to restore balance
between the rights and obligations of employees and those
of employers by
A. Permitting strikes in cases of health and safety
violations.
B. Outlawing the union shop.

A. The applicant who would be most productive for


the job should be hired.
B. The work force employed should be in the same
proportion as the demographics of the total
population.

C. Designating a list of unfair labor practices on the


part of employers.
D. Designating a list of unfair labor practices on the
part of unions.

C. Equal opportunity strengthens the seniority system.


D. Wages are inversely related to marginal
productivity.
[30] Source: CMA 1291 1-25

[34] Source: CMA 1291 1-30


The concept of comparable worth has become an
important issue in the U.S. in recent years. This concept
addresses the comparability of
A. U.S. wage rates with foreign wage rates adjusted

for differences in exchange rates.


B. Wage rates for different jobs requiring different
skills.
C. Wage rates for the same job in union and
non-union environments.
D. Wage rates for the same job in the government
and the private sector.
[35] Source: CMA 0693 1-11
The basic purpose of the Securities Exchange Act of 1934
was to

C. Compliance costs are a burden for small firms,


and Congress has chosen to exempt such firms.
D. The lobby for small businesses has been so active
that Congress has chosen to exempt such firms.
[39] Source: CMA 0693 1-20
The Clean Air Act of 1970 charged the Environmental
Protection Agency (EPA) with all of the following
responsibilities except
A. Setting standards without consideration for the
cost of compliance to manufacturers.

A. Regulate outstanding securities.


B. Regulate new issues of securities.

B. Nullifying the right of an individual citizen to bring


suit to enforce standards as such suits will now be
initiated by the EPA.

C. Protect investors from investment losses.


D. Ensure the public that stock market crashes like
that of 1929 would not occur again.

C. Setting standards to prevent damage to items such


as visibility, crops, and buildings.
D. Establishing minimum ambient air standards for the
entire country.

[36] Source: CMA 0693 1-14


The private placement of debt securities has all of the
following advantages over offerings to the general public
except that a private placement
A. Does not require Securities and Exchange
Commission regulation.
B. Can be completed in less time than public
offerings.

[40] Source: CMA 0693 1-21


The federal Consumer Product Safety Commission
(CPSC) was created in 1972 and administers an act
covering those consumer products not already regulated by
the federal government. Specifically, Section 15 of the Act
(CPSA) requires manufacturers to take corrective steps for
a product that fails to comply with a product safety rule or
contains a defect that could create a substantial product
hazard. This provision is included because

C. Has no flotation expenses.


D. Can include indentures that are "tailor-made" to
the borrower's needs.
[37] Source: CMA 0693 1-13
Blue-sky laws are
A. The detailed regulations of the Securities and
Exchange Commission designed to prevent fraudulent
or misleading security issues.
B. Regulations of the Securities and Exchange
Commission governing the treatment of goodwill in
the sale of securities.
C. State laws designed to prevent fraudulent or
misleading security issues.
D. Local laws regulating the issuance of securities for
companies whose operations affect the environment.
[38] Source: CMA 0693 1-19
Many social regulations exempt small business from their
application. This exemption has been enacted because
A. Small businesses have no significant impact on
jobs and the environment.
B. There are too many small businesses, and funds
provided are inadequate to regulate them effectively.

A. The CPSC is dependent on the manufacturers to


provide information on product safety.
B. Voluntary standards are preferred to compulsory
ones, and this pushes regulation back into the hands
of manufacturers.
C. It encourages the CPSC to apply cost-benefit
analysis in formulating and implementing consumer
product safety standards.
D. The manufacturer may be compelled to publicize
this defect to consumers and/or refund the purchase
price.
[41] Source: CMA 0693 1-22
In recent years, a number of consumer groups have used a
variety of methods to persuade companies to change
behavior. Which one of the following weapons has not
been used by consumer groups in recent years?
A. Boycotts or refusals to purchase from the
offending company.
B. Shareholder resolutions at annual shareholder
meetings to direct a company's board of directors to
do or not to do certain things.
C. Creation of a competing company to replace the
offending company.

D. Cooperation through meetings that attempt to find


common ground for action by both parties.
[42] Source: CMA 0693 1-23
Which one of the following is not a characteristic of the
Occupational Safety and Health Administration (OSHA)?
OSHA
A. Encourages labor-management committees to
formulate safety and health programs.
B. Inspections are primarily focused on health issues,
including long-term exposure to such substances as
asbestos and cotton dust.
C. Has the authority to levy monetary penalties on
non-compliant employers.
D. Inspections are primarily reactionary; that is, take
place after major injuries have occurred.
[43] Source: CMA 0693 1-24
Regulated industries, particularly those subject to
industry-specific economic regulation, have several
common characteristics. Which one of the following
characteristics is not associated with most regulated
industries?
A. These industries are capital intensive.
B. The social side effects or consequences of the
actions of these firms may be undesirable.

B. Market allocation or division of customers,


markets, or production.
C. Concerted refusals to deal or boycotts involving
any third party.
D. Tie-in sales or any effort to force a buyer to
purchase less desirable products in order to purchase
the desired product.

[46] Source: CMA 0693 1-27


The impact of successful prosecution for restraint of trade
violations of the Sherman Antitrust Act has been less than
many proponents hoped for because
A. Only a limited number of criminal prosecutions
have ever been filed under Section 1 of the Act.
B. Most companies avoid behaviors that might be
construed as antitrust violations because of the
likelihood of fines and even jail sentences.
C. The predominant criterion for criminal prosecution
is market share; a company's intent is inconsequential.
D. The courts created in the beginning a "rule of
reason" that required determining whether
competition was lessened by the company's behavior.

C. Such industries are often described as "affected


with a public interest" or considered vital industries.

[47] Source: CMA 0693 1-28


The first major federal equal job opportunity law since the
1970s is the Americans with Disabilities Act (ADA) of
1990. This act does not

D. Most of these industries are permitted to earn a


return that is equitable to all consumers.

A. Ban discrimination against people with disabilities


in employment.
B. Provide tax incentives for compliance costs.

[44] Source: CMA 0693 1-26


Antitrust suits, particularly civil rather than criminal suits,
may be brought by many different individuals or agencies.
Which one of the following does not have power to bring
such suits.
A. U.S. Department of Justice.
B. A state attorney general, when authorized by a
state antitrust law.
C. A private party, such as a competitor.
D. U.S. Department of Commerce.
[45] Source: CMA 0693 1-25
Supreme Court interpretations of the Sherman Act of 1890
have determined that certain types of agreements,
conspiracies, or combinations are in and of themselves so
restrictive of competition as to be conclusively presumed
unreasonable restraints of trade. These "per se" violations
do not involve the application of a "rule of reason" but are
considered illegal merely because they exist. Which one of
the following offenses is the most serious, i.e., which one
has produced the most criminal antitrust violations?
A. Price-fixing or tampering with the price structure.

C. Ban discrimination against people with disabilities


in transportation.
D. Provide federal funds for the implementation of
ADA by employers and those providing public
accommodation.
[48] Source: CMA 0693 1-29
In recent years, governmental bodies have tried a number
of devices to encourage environmental stewardship. At
least fourteen major federal acts or amendments to acts
were passed between 1969 and 1990. Which one of the
following is not a governmental device used in recent
years?
A. Direct regulation or legally enforceable
environmental quality standards.
B. Pollution charges or fees for undesirable waste
released by the firm.
C. Relocation of facilities to offshore sites where
governmental standards are less stringent.
D. Buying and selling of pollution rights involving
companies whose emissions are below the standard.

A. Make pollution illegal.


[49] Source: CMA 0693 1-30
Pharmaceutical companies must be in compliance with
regulations set forth by the Food and Drug Administration
(FDA). The FDA is responsible for all of the following
except
A. Allowing extensions to patent lives of
pharmaceutical companies in order to recover time
lost during the premarket FDA regulatory review.
B. Considering benefit-risk trade-offs when
evaluating the safety of drugs prior to approval.

B. Regulate the amount of pollution a firm can create.


C. Promote social awareness about the harm due to
pollution.
D. Allow marketable permits for pollution that firms
can buy or sell like any other commodity.
[54] Source: CMA 1293 1-6
A horizontal merger is a merger between

C. Maintaining purity standards and regulations


concerning drug composition and potency.

A. Two or more firms from different and unrelated


markets.

D. Approving drugs that lack substantial evidence


that they will have the effects that are represented in
advertisements.

B. Two or more firms at different stages of the


production process.
C. A producer and its supplier.

[50] Source: CMA 1293 1-1


The Sherman Antitrust Act
A. Is intended for international corporations.
B. Prohibits labor unions from forming closed shops.

D. Two or more firms in the same market.


[55] Source: CMA 1293 1-7
In unionized and nonunionized sectors of the economy,
labor unions generally

C. Outlaws all monopolies.

A. Result in wage equality in both sectors.

D. Discourages firms from monopolizing.

B. Raise wages in the unionized sector.


C. Raise wages at the same rate in both sectors.

[51] Source: CMA 1293 1-2


In a regulated industry such as electricity or gas,

D. Lower wages in both sectors.

A. The forces of demand and supply determine price.


B. Firms can enter or exit the industry easily.
C. Firms can decide on the quality of product or
service.

[56] Source: CMA 1293 1-8


In relation to labor unions, a closed-shop is where
A. Only those belonging to the union can work.
B. Belonging to the union is not necessary.

D. Regulatory bodies determine the quality and set


the price.

C. Collective bargaining is not permitted.


D. Workers have the right to work.

[52] Source: CMA 1293 1-4


The primary reason for social regulation (Clean Air Act,
Water Pollution Control Act, Food and Drug Act, etc.) is
that
A. The free market provides minimal safety and
environmental protection.
B. Consumer and environmental groups are politically
powerful.

[57] Source: CMA 1293 1-9


The most common form of business organization in terms of
number of firms in the United States is the
A. Partnership.
B. Limited partnership.
C. Sole proprietorship.

C. Social benefits from such regulations always


exceed the costs.

D. Corporation.

D. Social regulation is more desirable.


[53] Source: CMA 1293 1-5
An efficient way to control pollution in line with the
expectations of the Clean Air Act Amendments of 1990 is
to

[58] Source: CMA 1293 1-11


Environmental policies undertaken in the United States that
attempt to weigh the costs and benefits of environmental
improvement
A. Will result in the elimination of all pollution.

B. Cannot work because business firms are not


willing to cooperate.

A. Establish set profit percentages for firms in


regulated industries.
B. Prohibit firms in the same industry from engaging in
joint ventures.

C. Grant tax relief to the firms who pollute the least.


D. Would leave an acceptable amount of pollution.
[59] Source: CMA 1293 1-12
Commercial bank deposits are currently insured to
$100,000 per account by the
A. Federal Reserve System.

C. Ensure a free and competitive market in which


consumer demand dictates prices.
D. Limit competition to increase output and lower
prices.
[63] Source: Publisher
Pricing agreements are allowable in which of the following
situations?

B. Federal Saving and Loan Insurance Corporation.


A. An agreement to lower prices for consumers.
C. Federal Deposit Insurance Corporation.
D. U.S. Treasury.
[60] Source: Publisher
Which of the following statements about the National
Environmental Policy Act (NEPA) is most likely to be
false?
A. NEPA requires federal agencies to consider
certain potential environmental consequences in their
decision-making process.

B. An agreement to fix prices to eliminate unfair


competition.
C. An agreement to stabilize prices reasonably and
fairly for consumers.
D. Never.
[64] Source: Publisher
To be a monopoly, a firm must
A. Be the sole provider of the product.

B. NEPA allows the federal government to bring suit


against any private person who violates NEPA's
provisions.
C. Under NEPA, federal agencies do not have to
give environmental considerations priority over other
concerns in their decision-making processes.
D. NEPA augments the power of every existing
agency with respect to considering the environmental
consequences of their proposed actions.
[61] Source: Publisher
Which of the following statements is true with respect to the
Clean Water Act (CWA)?
A. Despite the CWA's prohibitions, it allows persons
to discharge pollutants into waters subject to its
jurisdiction as long as navigation thereon will not be
permanently obstructed.
B. The CWA subjects all bodies of water located in
the United States, whether flowing or not, to its
protection.
C. The notion of protecting waters within the
jurisdiction of the United States began with the
CWA.
D. The CWA seeks to restore and maintain the
physical and biological integrity of the waters of the
United States.

B. Have a market share of at least 80%.


C. Have the power to control prices or exclude
competition.
D. Have entered into contracts, combinations, or
conspiracies in restraint of trade.
[65] Source: Publisher
Which of the following firms are subject to the requirements
of the Foreign Corrupt Practices Act?
A. All corporations that engage in intrastate or
interstate commerce.
B. All corporations required to be registered under
the Securities Act of 1933.
C. All corporations required to be registered under
the Securities Exchange Act of 1934.
D. All corporations incorporated in the U.S.
[66] Source: Publisher
A major result of the Foreign Corrupt Practices Act is that
corporations are now required to
A. Keep accurate accounting records and maintain
an internal control structure.
B. Prepare financial statements in accordance with
U.S. and international accounting standards.

[62] Source: Publisher


Antitrust laws are intended to

C. Produce information to the SEC on foreign


commerce and foreign political party information.

business.
D. Adhere to U.S. auditing standards.
[67] Source: Publisher
The Foreign Corrupt Practices Act prohibits
A. Bribes to all foreigners.
B. Small bribes to foreign officials that serve as
facilitating or grease payments.
C. Bribery only by corporations and their
representatives.

[72] Source: CMA 0694 1-16


The Sherman Antitrust Act of 1890 prohibits
A. Price discrimination, tying contracts,
anti-competitive mergers, and interlocking
directorates.
B. Unfair competition and deceptive business
practices.
C. Mergers without prior notification to the Justice
Department.

D. Bribes to foreign officials to influence official acts.


D. Restraint of trade and monopoly.
[68] Source: CIA 0594 IV-67
Which of the following is not considered to be an
advantage of organizing a business as a sole
proprietorship?

[73] Source: CMA 0694 1-17


The Taft-Hartley Act of 1947 was an attempt to restore
balance between the rights of employers and those of
employees by

A. Easy and inexpensive to organize.


A. Outlawing the union shop.
B. Allows freedom of action for the entrepreneur.
C. Provides strong incentives to manage the business
efficiently.
D. Allows the proprietor to carry out all basic
management functions.

B. Imposing mandatory binding arbitration on both


employers and unions.
C. Designating certain labor practices on the part of
unions as unfair and outlawing the closed shop.
D. Permitting strikes on the part of federal
employees.

[69] Source: CIA 0594 IV-66


In which legal form of business organization do the owners
of the business enjoy limited liability?
A. Sole proprietorship.
B. Partnership.

[74] Source: CMA 0694 1-18


The Robinson-Patman Act of 1936 prohibits price
discrimination unless
A. The price is set to meet a competitor's price
quote.

C. Corporation.
D. Monopoly.
[70] Source: CIA 0589 IV-47
Which of the following represents a disadvantage of the
partnership form of business organization?

B. The price reflects cost increases caused by


government specification.
C. The price is set in a market characterized by
monopolistic competition.
D. Specific prior approval is obtained from the
Justice Department.

A. Unlimited liability.
B. Double taxation.
C. Complexity in formation.

[75] Source: CMA 1294 1-10


Which one of the following antitrust laws prohibits price
discrimination, tying contracts, anticompetitive mergers, and
interlocking directorates?

D. Regulatory requirements.
A. Sherman Antitrust Act.
[71] Source: CIA 0593 IV-47
A limited partnership is typically used to limit the

B. Clayton Antitrust Act.


C. Antitrust Improvements Act.

A. Withdrawals of individual partners.


D. Federal Trade Commission Act.
B. Duration of the partnership.
C. Liabilities of some of the partners.
D. Authority of general partners to manage the

[76] Source: CMA 1294 1-11


The National Labor Relations Act of 1935, also known as
the Wagner Act,

A. Provides for compulsory binding arbitration in the


case of national emergency.

C. Covered as determined by the Department of


Justice.

B. Guarantees the right of self-organization and the


right to collective bargaining with management.

D. Covered to the same extent as any other industry.

C. Designates a list of unfair labor practices on the


part of unions.

[81] Source: CMA 1295 1-21


Social regulation is often criticized by industry as inefficient.

D. Requires that union officials be elected on a


regular basis and that secret ballots be used.

Firms perceive this inefficiency to be a result of


A. The failure to consider the marginal benefits
relative to the marginal costs.

[77] Source: CMA 1294 1-12


The Labor-Management Relations Act, also known as the
Taft-Hartley Act of 1947,
A. Prohibits employers from interfering with the right
of employees to form unions.
B. Obligates employers to bargain in good faith with
unions.
C. Outlaws yellow-dog contracts, which require
employees to remain as nonunion members.
D. Prohibits unions from coercing employees to
become union members.
[78] Source: CMA 1294 1-13
The Sherman Antitrust Act
A. Created the Federal Trade Commission.
B. Did not establish penalties for offenders of the
Act.
C. Prohibits mergers and acquisitions that
substantially lessen competition.
D. Declares illegal, attempts at restraint of trade.
[79] Source: CMA 1294 1-14
Social legislation, as exemplified by the Occupational
Safety and Health Act and Environmental Protection Act, is
frequently criticized for being inefficient because the
agencies
A. Use flexible rather than rigid standards.
B. Impose compliance costs that are low compared
to the benefits received by society.
C. Rely heavily on the free market to allocate
resources.
D. Rarely consider the marginal benefits relative to
the marginal costs.
[80] Source: CMA 1295 1-20
With respect to federal antitrust laws, regulated industries
are
A. Completely exempt.
B. Covered as determined by statute and the courts.

B. Lenient enforcement policies.


C. Concern for the quality of life but not the quality of
products.
D. The use of the internal revenue tax code instead of
strict compliance penalties.
[82] Source: CMA 1295 1-22
Which one of the following transactions would be
considered a violation of the Robinson-Patman Act?
A. The sale of goods of like quality at different prices
to two different wholesalers, both of whom are
located outside the United States.
B. The sale of goods of like quality within the United
States at different prices based on cost differences
related to the method of delivery.
C. The sale of goods of like quality within the United
States at different prices to two different wholesalers;
all parties are located within the same state.
D. The sale of goods of like quality within the United
States but across state lines at different prices to two
different wholesalers in the same geographic area.
[83] Source: CMA 1295 1-23
Which one of the following examples of corporate behavior
would most clearly represent a violation of the Sherman
Act?
A. A retailer offers quantity discounts to large
institutional buyers.
B. The members of a labor union meet and agree not
to work for a specific firm unless the starting wage is
at least $10 per hour.
C. Two firms that are in different, unrelated industries
merge.
D. Two firms in the same industry agree in a
telephone conversation to submit identical bids on a
government contract.
[84] Source: CMA 1295 1-24
The Clayton Act, as amended, prohibits all of the following
except
A. Tying contracts that require a customer who is

buying one product to buy a related but perhaps


unwanted product.
B. Price discrimination by sellers.
C. Interlocking directorates in large competing
organizations.

II. National Highway Traffic Safety Administration.


III. Consumer Product Safety Commission.
IV. Occupational Safety and Health Administration.
V. Environmental Protection Agency.
Which of the above agencies are considered social
agencies?
A. I, II, and IV only.

D. Unfair and deceptive business practices, such as


misleading advertising.

B. I and II only.
C. I, II, and III only.

[85] Source: CMA 1295 1-25


The acquisition of a retail shoe store by a shoe
manufacturer is an example of
A. Vertical integration.
B. A conglomerate.
C. Market extension.

D. All of the agencies.


[90] Source: CMA 1295 1-30
The two major functions of the Federal Trade Commission
are
A. Antitrust actions and the regulation of foreign
trade.

D. Product extension.
B. Import quality inspections and anti-dumping
measures.
[86] Source: CMA 1295 1-26
Blue sky laws are
C. Antitrust actions and consumer protection.
A. Federal laws that make it unlawful to use
deceptive practices in the sale of securities.
B. Federal laws that limit the amount of air pollution
in a specific geographic area.
C. State laws that regulate the sale of securities.

D. Price discrimination and unfair trade practices.


[91] Source: CMA 0685 1-34
Membership in labor unions as a proportion of the total
work force has declined in the past decade. The reason
most often given for this decline is that

D. State laws that regulate the environment.


A. The expansion in employment has occurred
primarily in service industries.
[87] Source: CMA 1295 1-27
Which one of the following federal acts requires unions to
retain financial records and submit financial reports to
federal authorities?

B. Unions have been less aggressive in organizing.


C. Workers have become disenchanted with union
leaders.

A. Taft-Hartley Act of 1947.


B. Wagner Act of 1935.

D. A larger proportion of union members have


reached retirement age and the replacement workers
are not joining the union.

C. Securities Exchange Act of 1934.


D. Landrum-Griffin Act of 1959.
[88] Source: CMA 1295 1-28
Which one of the following is not exempted from federal
antitrust regulation?

[92] Source: CMA 0685 1-35


Business organizations find that union behavior is changing
due in part to the change in composition of union
membership. A change that is not occurring in union
membership is that on the average there are more
A. Members from minority groups.

A. Labor unions.
B. Women members.
B. Intrastate commerce.
C. Members from blue-collar jobs.
C. Patents and copyrights.
D. Members from the public sector.
D. Telecommunications companies.
[89] Source: CMA 1295 1-29
Listed below are five federal agencies.
I. Food and Drug Administration.

[93] Source: Publisher


Corporate social responsibility is
A. Effectively enforced through the controls
envisioned by classical economics.

B. Defined as the obligation to shareholders to earn a


profit.

A. Outlaws interlocking directorates in large


competing corporations.

C. More than the obligation to shareholders to earn a


profit.

B. Prohibits price discrimination by sellers.

D. Defined as the obligation to serve long-term,


organizational interests.

C. Forbids tying contracts that require a purchaser to


buy an unwanted product to get another.
D. Forbids unfair and deceptive business practices,
such as misleading advertising.

[94] Source: Publisher


A common argument against corporate involvement in
socially responsible behavior is that
A. It encourages government intrusion in decision
making.

[99] Source: CMA 0696 1-24


If a product is believed to pose an imminent hazard to
consumers, the Consumer Product Safety Commission
(CPSC)

B. As a legal person, a corporation is accountable for


its conduct.

A. May seek a court order to have the product


banned or seized.

C. It creates goodwill.

B. May ban or seize the product upon a majority


vote by the commissioners of the CPSC.

D. In a competitive market, such behavior incurs


costs that place the company at a disadvantage.
[95] Source: CMA 0696 1-18
Resale price maintenance is an example of

C. May ban or seize the product, but only after a


notice has been published in the Federal Register for
30 days.
D. Can only ask the manufacturer voluntarily to
withdraw or recall the product.

A. Horizontal price fixing.


B. Vertical price fixing.
C. Preemptive buying.

[100] Source: CMA 0696 1-25


Which one of the following questions can be asked by an
employer during an employment interview ensuring
compliance with anti-discrimination rules?

D. Tying arrangements.
A. "How old are you?"
[96] Source: CMA 0696 1-21
A main provision of the Securities Act of 1933, as
amended in 1934, is the requirement that

B. "Do you have a disability?"

A. Bonds be issued only under a trust indenture


approved by the Securities and Exchange
Commission (SEC).

D. "Do you have references?"

B. Public utility holding companies register with the


SEC.
C. New securities offered for sale in interstate
commerce be registered with the SEC.

C. "Are you married?"

[101] Source: CMA 0696 1-26


A factory moves from a non-right-to-work state to a
right-to-work state, and the union continues to represent
the employees. In the right-to-work state,
A. The employees can be forced to join the union
before they start to work.

D. All security brokers be licensed by the SEC.


B. The employees can be forced to join the union
after thirty days on the job.
[97] Source: CMA 0696 1-22
The Clayton Act of 1914 prohibits

C. The employees can decide not to be involved with


the union.

A. Closed-shop labor unions.


B. Sellers' price discrimination.

D. The employees can be forced to make payments


to the union.

C. Group boycotts.
D. Oligopolies.
[98] Source: CMA 0696 1-23
The Federal Trade Commission Act, as amended,

[102] Source: CMA 0696 1-27


Which one of the following approaches to regulation best
describes mandatory labeling information concerning food
content and nutritional values?
A. Unregulated markets subject to antitrust

enforcement.
B. Disclosure of all relevant information.

A. Requires pre-approval from the Federal Trade


Commission for interlocking directorates.

C. Changes in liability rules toward a stricter liability.

B. Prohibits price discrimination.

D. Creation of marketable property rights.

C. Requires notification prior to a merger.


D. Prohibits restraints of trade and monopoly.

[103] Source: CMA 0696 1-28


Which one of the following forms of law is created when
regulatory agencies transform statutes into regulations and
enforcement procedures?

[108] Source: CMA 1296 1-24


Sunset laws refer to

A. Constitutional law.

A. Protection of employees nearing retirement.

B. Statutory law.

B. Federal laws that prohibit the sale of securities for


companies that have not been in business for at least
3 months.

C. Administrative law.
D. Judicial law.
[104] Source: CMA 1296 1-20
A franchisor's requirement that its franchisees buy inputs
from a particular supplier is
A. Illegal according to the provisions of the
Robinson-Patman Act.

C. State laws that prohibit the sale of alcohol on


Sunday.
D. A periodic review of, and a fixed termination date
for, government programs.
[109] Source: CMA 1296 1-25
All of the following are legal rights of shareholders in U.S.
publicly traded companies except the right to

B. Illegal according to the principles of common law.


A. Vote on major mergers and acquisitions.
C. Legal, but the franchisor must receive approval
from the Federal Trade Commission.
D. Legal as long as such a requirement is necessary
to assure product quality.

B. Receive dividends if declared.


C. Vote on charter and bylaw changes.
D. Vote on major management changes.

[105] Source: CMA 1296 1-21


A basic purpose of the securities laws in the United States
is to regulate the issuance of investment securities by

[110] Source: CMA 1296 1-29


The Americans with Disabilities Act (ADA) of 1990

A. Requiring disclosure of all relevant information so


that investors can make informed decisions.

A. Permits employers to inquire about a job


applicant's prior health insurance claims.

B. Prohibiting the issuance of non-investment grade


securities.

B. Bans discrimination against employees with


physical disabilities, but not those with mental
disabilities.

C. Ensuring that all shareholders have an equal vote


in the election of a board of directors.

C. Requires organizations with 25 or more

D. Providing a regulatory framework for those states


that do not have their own securities laws.

employees to provide reasonable accommodation for


employees with disabilities.

[106] Source: CMA 1296 1-22


The Sherman Antitrust Act
A. Established the Federal Trade Commission.
B. Prohibits collective boycotts.

D. Provides federal funds to employers who


implement its provisions.
[111] Source: CMA 1296 1-30
The Insider Trading and Securities Fraud Enforcement Act
of 1988 in conjunction with the Insider Trading Sanctions
Act of 1984

C. Prohibits price discrimination.


D. Established the concept of a patent.

A. Enacted monetary penalties for insider trading for


the first time.
B. Increased monetary penalties for insider trading.

[107] Source: CMA 1296 1-23


The Antitrust Improvements Act of 1976

C. Enacted criminal penalties for market manipulation

and securities fraud.

rules.

D. Empowered the Securities and Exchange


Commission with the authority to recover gains
resulting from the illegal use of insider information.

C. Regulatory rules being rigid and hard to change.

[112] Source: CMA 0697 1-22


A large public company that is well-known can reduce the
time required to register and issue securities by using a(n)

D. Regulations increasing the cost of products and


services.
[117] Source: CMA 0697 1-29
All the following are functions of the Securities and
Exchange Commission except the

A. Shelf registration.
A. Review of stock trades by corporate insiders.
B. Indenture agreement.
C. Secondary market registration.
D. Red herring registration.
[113] Source: CMA 0697 1-25
Which one of the following antitrust laws prohibits price
discrimination and other exclusionary practices that may
give certain firms a competitive advantage over other firms
in the same market?
A. Celler-Kefauver Act.
B. Federal Trade Commission Act.
C. Sherman Act.
D. Robinson-Patman Act.

B. Regulation of interstate offerings of new securities


to the public.
C. Setting of rules concerning the proxy process of
large public companies.
D. Determination of fair trading prices for the
common stock of large public companies.
[118] Source: Publisher
The Comprehensive Environmental Response,
Compensation, and Liability Act (CERCLA), also
generically known as the "Superfund," applies to the release
of "hazardous substances." Which of the following is
statutorily included in the definition of the term "hazardous
substance"?
A. Crude oil.
B. Gasoline.

[114] Source: CMA 0697 1-26


Which one of the following laws addresses the issue of
insider trading?

C. Asbestos.
D. Natural gas.

A. Federal Trade Commission Act.


B. Securities Exchange Act.
C. Clayton Act.

[119] Source: Publisher


Which of the following statements about the National
Environmental Policy Act (NEPA) is most likely to be
incorrect?

D. North American Free Trade Agreement.


[115] Source: CMA 0697 1-27
Airlines headquartered in the United States have
experienced deregulation in many aspects of their business.
Which one of the following aspects of the business is still
subject to regulation by the federal government?
A. On-time departure standards.
B. Domestic fare schedules.
C. Collective bargaining.
D. International fare schedules.
[116] Source: CMA 0697 1-28
All of the following are criticisms generally made of federal
regulatory agencies and policies except
A. Taxes imposed by agencies being too high.
B. Political considerations influencing policies and

A. NEPA requires federal agencies to consider


environmental consequences in their decision-making
process.
B. NEPA allows the federal government to bring suit
against any private person who violates NEPA's
provisions.
C. Under NEPA, federal agencies do not have to
give environmental considerations priority over other
concerns in their decision-making processes.
D. NEPA augments the power of existing agencies
with respect to considering environmental
consequences of proposed actions.
[120] Source: Publisher
Which of the following statements regarding the Clean
Water Act (CWA) is correct?
A. It allows persons to discharge pollutants into
waters subject to its jurisdiction as long as navigation
thereon will not be permanently obstructed.

making.
B. The CWA subjects all bodies of water located in
the United States, whether flowing or not, to its
protection.
C. The notion of protecting waters within the
jurisdiction of the United States began with the
CWA.
D. The CWA seeks to restore and maintain the
physical and biological integrity of the waters of the
United States.
[121] Source: CMA 1294 1-9
(Refer to Figure 4.) One way of measuring government
performance with respect to antitrust regulation is
cost-benefit analysis. In the graph, P1 and Q1 represent
the price and quantity under a monopoly. Effective
government regulation could lower the price to P2 which
could increase the quantity to Q2. On the graph, the area
ABC represents the

B. As a legal person, a corporation is accountable for


its conduct.
C. It creates goodwill.
D. In a competitive market, such behavior incurs
costs that place the company at a disadvantage.
[125] Source: Publisher
Integrity is an ethical requirement for all financial
managers/management accountants. One aspect of integrity
requires
A. Performance of professional duties in accordance
with applicable laws.
B. Avoidance of conflict of interest.
C. Refraining from improper use of inside
information.

A. Total cost of producing the additional output.


B. Excess unsatisfied demand caused by the
regulation.
C. Efficiency gain from antitrust action.
D. Total benefit generated by the change.

D. Maintenance of an appropriate level of


professional competence.
[126] Source: Publisher
Under the express terms of the IMA Code of Ethics, a
financial manager/management accountant may not
A. Advertise.

[122] Source: Publisher


Sheila is a financial manager who has discovered that her
company is violating environmental regulations. If her
immediate superior is involved, her appropriate action is to
A. Do nothing since she has a duty of loyalty to the
organization.
B. Consult the audit committee.
C. Present the matter to the next higher managerial
level.
D. Confront her immediate superior.
[123] Source: Publisher
Corporate social responsibility is
A. Effectively enforced through the controls
envisioned by classical economics.

B. Encroach on the practice of another financial


manager/management accountant.
C. Disclose confidential information unless authorized
or legally obligated.
D. Accept other employment while serving as a
financial manager/management accountant.
[127] Source: CMA 1
According to Statements on Management Accounting
Number 1C (SMA 1C) (revised), Standards of Ethical
Conduct for Practitioners of Management Accounting and
Financial Management, a practitioner has a responsibility to
recognize professional limitations. Under which standard of
ethical conduct would this responsibility be included?
A. Competency.
B. Confidentiality.

B. Defined as the obligation to shareholders to earn a


profit.
C. More than the obligation to shareholders to earn a
profit.
D. Defined as the obligation to serve long-term,
organizational interests.
[124] Source: Publisher
A common argument against corporate involvement in
socially responsible behavior is that
A. It encourages government intrusion in decision

C. Integrity.
D. Objectivity.
[128] Source: CMA 2
At Key Enterprises, the controller is responsible for
directing the budgeting process. In this role, the controller
has significant influence with executive management as
individual department budgets are modified and approved.
For the current year, the controller was instrumental in the
approval of a particular line manager's budget without
modification, even though significant reductions were made
to the budgets submitted by other line managers. As a

token of appreciation, the line manager in question has


given the controller a gift certificate for a popular local
restaurant. In considering whether or not to accept the
certificate, the controller should refer to which section of
Statements on Management Accounting Number 1C
(SMA 1C) (revised), Standards of Ethical Conduct for
Practitioners of Management Accounting and Financial
Management?
A. Competency.
B. Confidentiality.
C. Integrity.
D. Objectivity.

CMA PART 1 D
Domestic Institutional
Environment of Business
ANSWERS

Answer (C) is incorrect because direct corporate


contributions are not allowed. However,
corporations, trade associations, and unions may
form PACs and encourage employees to participate,
and company funds may be used to organize and
administer the PAC.
Answer (D) is incorrect because individuals may give
funds either directly to candidates or to PACS.

[1] Source: CMA 0685 1-31


Answer (A) is incorrect because it is not meaningful
in this context.
Answer (B) is incorrect because it is not meaningful in
this context.
Answer (C) is incorrect because it is not meaningful
in this context.
Answer (D) is correct. Sunset laws provide for a
periodic legislative review of and a fixed termination
date for regulatory acts and agency enabling statutes.
The legislative body must take affirmative action to
determine a program's usefulness and to reauthorize
it. The objective is to avoid the creation of programs
that continue after their purposes have been achieved.
[2] Source: CMA 1286 1-23
Answer (A) is incorrect because all private
corporations are created under state statutes since no
federal incorporation statute exists.
Answer (B) is correct. Corporations do not exist at
common law. They are entirely creatures of statutory
law. In the U.S., private corporations are chartered
by the individual state governments.
Answer (C) is incorrect because all private
corporations are created under state statutes since no
federal incorporation statute exists.
Answer (D) is incorrect because a private
corporation can only come into being if the state
incorporation statute is complied with. The
corporation must exist before directors can be
elected.
[3] Source: CMA 1286 1-28
Answer (A) is correct. Federal law prohibits business
corporations from donating to political election
campaigns at the federal level. Note that this does not
prohibit donations to state election campaigns,
although some states have such laws. Political action
committees (PACs) have therefore grown in
importance over the past decade as a means of
circumventing this prohibition.
Answer (B) is incorrect because direct corporate
contributions are not allowed. However,
corporations, trade associations, and unions may
form PACs and encourage employees to participate,
and company funds may be used to organize and
administer the PAC.

[4] Source: CMA 1286 1-26


Answer (A) is incorrect because the concept of
negligence requires proof of fault.
Answer (B) is incorrect because the theory of strict
liability requires neither a warranty nor even privity of
contract.
Answer (C) is incorrect because contributory
negligence is not a defense in a strict liability case, but
comparative negligence may be pleaded in some
states.
Answer (D) is correct. Strict liability may be imposed
in product liability cases although the defendant was
not at fault. In a suit against the manufacturer, a
plaintiff must prove that the product was defective,
the defect rendered it unreasonably dangerous to a
user, the seller was engaged in the business of selling
the particular product, and the product reached the
user without substantial change from the condition in
which it was sold. However, absolute, not strict,
liability is imposed for damages caused by inherently
dangerous products.
[5] Source: CMA 0687 1-13
Answer (A) is incorrect because antitrust laws do not
regulate prices. However, prices may not be
discriminatory or collusive.
Answer (B) is incorrect because collecting price
information is lawful.
Answer (C) is incorrect because antitrust laws apply
to all firms engaged in interstate commerce,
regardless of size.
Answer (D) is correct. The Sherman Antitrust Act of
1890 was the first U.S. antitrust law. It prohibited
contracts, combinations, and conspiracies in restraint
of trade and made monopolizing or attempting to
monopolize any part of interstate commerce a
criminal offense. Allocating customers among firms
within an industry, for example, by division of
markets, is a per se violation of the Act.
[6] Source: CMA 0687 1-15
Answer (A) is correct. Tie-in sales involve a
seller/lessor's requirement that the buyer/lessee
purchase/lease another distinct product in order to
obtain the first. They are generally prohibited under
the Clayton Act of 1914 unless a small company is
attempting to enter a market or the tie-in is needed to
protect the firm's goodwill (e.g., by maintaining a

certain quality standard).


Answer (B) is incorrect because tie-in sales
requirements are generally illegal if the seller has
enough market power to restrict competition.
Answer (C) is incorrect because tie-in sales
requirements are generally illegal if the seller has
enough market power to restrict competition.
Answer (D) is incorrect because tie-in sales
requirements are generally illegal if the seller has
enough market power to restrict competition.
[7] Source: CMA 0687 1-22

However, this prohibition does not apply if such


contracts are designed to assure product quality.
Answer (B) is incorrect because these contracts are
not always illegal, and patents have nothing to do with
these provisions.
Answer (C) is incorrect because price discrimination
is prohibited under the Robinson-Patman Act of
1936.
Answer (D) is incorrect because exclusive dealing is
generally prohibited under the Clayton Act.
[10] Source: CMA 1287 1-17

Answer (A) is incorrect because the FCC allocates


access to the radio spectrum to those who will best
use the medium.

Answer (A) is incorrect because profits and prices


are not set by antitrust laws other than to the extent
that price discrimination is prohibited.

Answer (B) is correct. The FCC is an independent


agency formed in 1934 to regulate all methods of
communications over the public airwaves. The
functions of the FCC include allocating transmission
frequencies, issuing transmission licenses, fostering
effective and efficient use of communication
resources, and regulating common communications
carriers engaged in interstate and foreign operations.
Among the rules is a limitation on the number of
television stations that can be owned by a television
network.

Answer (B) is incorrect because firms may enter into


joint ventures with foreign firms.

Answer (C) is incorrect because profits are not


regulated. Radio and TV station owners may make
whatever profits they can as long as the airwaves are
used consistently with "the public interest,
convenience, and necessity."

Answer (C) is incorrect because patents are available


to all inventors, regardless of size.
Answer (D) is correct. Antitrust laws are designed to
promote more efficient allocation of resources,
greater choice for consumers, greater business
opportunities, fairness in economic behavior, and
avoidance of concentrated political power resulting
from economic power. Competition results in greater
output and lower prices than other market structures.
Agreements to limit output of a product would lessen
competition and increase the price to consumers.
[11] Source: CMA 1287 1-18

Answer (D) is incorrect because program content is


not censored (but inappropriate programming can
result in future loss of license).
[8] Source: CMA 1287 1-14
Answer (A) is incorrect because tariffs are to protect
businesses, not society as a whole.
Answer (B) is correct. A social regulation is one for
the good of society as a whole. Its purpose is to
reach behavior that is thought to be not purely
financial or economic. The prohibition of
discrimination in employment and housing are
examples.
Answer (C) is incorrect because patent laws are to
encourage new product development and protect
inventors.
Answer (D) is incorrect because such rules regulate
economic activity.

Answer (A) is incorrect because professionals and


their organizations are not exempt from the antitrust
laws.
Answer (B) is incorrect because agreements among
competitors to increase fees are illegal per se.
Answer (C) is incorrect because fee discrimination is
not illegal per se since many factors influence the fees
that a particular professional might charge. For
example, a professional might charge a lower price if
a service is to be performed during a normally slow
period.
Answer (D) is correct. Lawyers, public accountants,
and other professionals have received scrutiny from
the FTC in recent years concerning pricing policies.
In general, professionals cannot make agreements
that provide for either minimum or maximum fees
since price fixing by collusion among competitors is a
per se violation of the Sherman Act.
[12] Source: CMA 1287 1-19

[9] Source: CMA 1287 1-16


Answer (A) is correct. The Clayton Act prohibits
exclusive dealing arrangements in which the effect
may be to substantially lessen competition or tend to
create a monopoly in any line of commerce.

Answer (A) is incorrect because the Commerce


Department has no authority to enforce the antitrust
laws.
Answer (B) is incorrect because the Justice

Department also has the authority to enforce antitrust


laws.
Answer (C) is correct. The FTC, in conjunction with
the antitrust division of the Justice Department, has
broad authority to enforce the antitrust laws. Since
mergers may lessen competition or tend to create a
monopoly under the terms of the Clayton Act, they
are scrutinized by the FTC and the Justice
Department.
Answer (D) is incorrect because prohibitions against
mergers have not been repealed.
[13] Source: CMA 1287 1-20
Answer (A) is incorrect because users who receive
different volumes of service usually pay different
rates.
Answer (B) is incorrect because rates are also based
on other than original cost when facilities are acquired
by one utility from another utility.
Answer (C) is incorrect because utilities are
monopolies and are not permitted to refuse to deal
with paying customers.
Answer (D) is correct. Public utilities are natural
monopolies subject to governmental price controls.
State utility commissions establish utility rates by
setting rates so that capital providers earn a
reasonable return on their investment (the rate base).
The rate base is the amount of assets minus
depreciation used in the company's operations.
[14] Source: CMA 1287 1-21
Answer (A) is incorrect because an increase in
variable costs would decrease the number of firms in
the industry.
Answer (B) is incorrect because fixed costs will be
unaffected.
Answer (C) is incorrect because prices will increase
in the long run if marginal cost increases.
Answer (D) is correct. An increase in variable costs
will cause an increase in prices in both the short and
long run since the variable costs will have to be
covered in both periods. If only fixed costs were
increased, only the long run might be affected.

Answer (B) is incorrect because the FTC does not


regulate import quotas. Its operations are purely
domestic.
Answer (C) is incorrect because the Interstate
Commerce Commission regulates railroads and the
Federal Aviation Administration regulates airlines.
Answer (D) is incorrect because labor unions are
monitored by the Department of Labor.
[16] Source: CMA 1289 1-24
Answer (A) is incorrect because it is a provision of
the Taft-Hartley Act.
Answer (B) is correct. The Labor-Management
Relations Act of 1947, or Taft-Hartley Act, placed
restraints on unions that resulted in a more even
balance of power between labor and management.
The Act provides for mediation of disputes, prohibits
closed shops, encourages the individual states to pass
right-to-work laws, defines unfair labor practices,
and provides for an 80-day cooling-off period. It
does not guarantee workers the right to organize.
This right is guaranteed by the Wagner Act of 1935.
Answer (C) is incorrect because it is a provision of
the Taft-Hartley Act.
Answer (D) is incorrect because it is a provision of
the Taft-Hartley Act.
[17] Source: CMA 1289 1-29
Answer (A) is incorrect because federal law applies
in all states and supplements state blue-sky
(securities) laws.
Answer (B) is correct. The basic purpose of the
federal securities laws is to provide disclosure of
adequate information so that investors can evaluate
investments. This is accomplished through complex
registration and reporting requirements concerning the
issuance and subsequent trading of securities.
However, the federal government does not assess the
merits of these securities.
Answer (C) is incorrect because the SEC does not
determine the merits of securities; it evaluates whether
sufficient information is provided.
Answer (D) is incorrect because the securities laws
are not intended to influence the investment of capital
in more socially or economically beneficial ways.

[15] Source: CMA 1289 1-26


Answer (A) is correct. The Federal Trade
Commission Act of 1914 created the Federal Trade
Commission (FTC) to help provide day-to-day
enforcement of the antitrust laws. In addition to
having jurisdiction to enforce all of the antitrust laws,
the FTC has broad authority to prevent unfair
methods of competition and unfair or deceptive acts
or practices. The FTC covers virtually all aspects of
commerce. Any form of commercial deception is of
concern to the FTC.

[18] Source: CMA 1289 1-30


Answer (A) is correct. An administrative agency is a
public board, commission, officer, etc. (other than a
judicial or legislative body), with limited power to
enforce the law, make rules, and adjudicate disputes
involving private rights and duties. It may be
independent (the SEC, FTC, FCC) or executive
(OMB, cabinet departments). These agencies affect
almost all aspects of the nation's social, economic,
commercial, and political life. The activity of an

administrative agency is an exception to the


separation of powers doctrine because the agency
has the power to enforce the law (executive power),
make rules and regulations (legislative power), and
decide disputes (judicial power). Because an agency
is not a formally separate branch of government, it
exercises only that power delegated to it, and any act
outside the scope of the delegation is unconstitutional.
Answer (B) is incorrect because the GAO is the audit
arm of Congress and has no approval authority.
Answer (C) is incorrect because proposed
regulations are made public in the Federal Register.
Answer (D) is incorrect because there is no 1-year
notification requirement.

total amount of payments must be disclosed.


Open-end credit transactions, such as those involving
credit cards, also have specific, detailed disclosure
requirements.
[21] Source: CMA 0690 1-6
Answer (A) is incorrect because appropriate tests
can be given to all applicants.
Answer (B) is incorrect because the tests are not
legal if they are not directly related to job
requirements.
Answer (C) is incorrect because the tests are legal if
related to job requirements. Professionally developed
tests that meet EEOC guidelines are most likely to
survive scrutiny.

[19] Source: CMA 0690 1-2


Answer (A) is incorrect because the Consumer
Product Safety Act of 1972 exempts certain
products, e.g., aircraft, motor vehicles, food, drugs,
and cosmetics.
Answer (B) is incorrect because a ban on a product
is appropriate only if specific standards will be
ineffective.
Answer (C) is incorrect because industry standards
are encouraged.

Answer (D) is correct. The equal employment


opportunity laws are designed to prohibit
discrimination in employment. Employers do,
however, have the right to use selection procedures in
hiring as long as the procedures are not
discriminatory. Thus, an employer will have a defense
under Title VII of the Civil Rights Act of 1964 if it
uses professionally developed pre-employment ability
tests that are directly related to job requirements.
[22] Source: CMA 0691 1-21

Answer (D) is correct. The CPSC promotes


voluntary safety standards, develops and enforces
mandatory standards, prohibits unsafe products if
safety standards will not be sufficient, recalls
hazardous products, furnishes information to

Answer (A) is incorrect because the EEOC deals


mostly with large companies.

consumers, and works with state and local


governments. Thus, if a product is hazardous and
presents an unreasonable risk of injury and safety
standards will not adequately protect the public, the
CPSC may issue an appropriate order banning the
product. If a hazard is deemed to be imminent, it may
seek a U.S. District Court's authorization to seize the
product or halt its distribution.

Answer (C) is incorrect because the EEOC attempts


to conciliate disagreements on matters within its
authority, but it has no major policy not to seek legal
remedies, such as court orders, damage awards, or
loss of government contracts.

[20] Source: CMA 0690 1-5

Answer (B) is incorrect because the EEOC is not


concerned with employee productivity.

Answer (D) is correct. The EEOC has set goals and


timetables for employers to bring female and minority
work forces up to the appropriate percentages as
they relate to the available labor pool. The means of
achieving these goals, such as affirmative action
programs, are often highly controversial.

Answer (A) is incorrect because the Consumer


Credit Protection Act merely requires disclosure. It
does not regulate interest rates.
Answer (B) is incorrect because the Act did not
prohibit the use of appraisal fees.
Answer (C) is incorrect because the Act had nothing
to do with wage garnishment.
Answer (D) is correct. The Truth-in-Lending Act
applies to creditors that extend consumer credit to
individual debtors (not organizations) in amounts of
$25,000 or less. For a closed-end credit transaction,
e.g., the typical car loan, the total finance charge,
annual percentage interest rate, amount financed, late
charges, security interest held by the creditor, the
number and amounts of payments, due dates, and the

[23] Source: CMA 0691 1-24


Answer (A) is incorrect because fair trade laws
pertain to the prices charged by retailers.
Answer (B) is incorrect because the term fair
employment laws is nonsensical in this context.
Answer (C) is incorrect because the term open shop
laws is not customarily used to describe laws that bar
conditioning employment upon union membership.
Answer (D) is correct. State right-to-work laws are
expressly permitted by the NLRA. They allow
individuals to work for a unionized employer without
being compelled to join the union or pay dues.
Moreover, unions must represent all employees in the

bargaining unit, including these free riders.

Answer (A) is incorrect because the airline industry


has been deregulated.

sliding tax charge based on the amount of effluent


emitted. This method is preferred because, as effluent
discharge increases, a company's tax increases,
providing a strong incentive for firms to discover new
methods of controlling pollution. Rather than dictating
technology, the tax allows firms to seek out the
technology that is the most cost effective.

Answer (B) is incorrect because the banking industry


has been deregulated.

Answer (C) is incorrect because the EPA would not


be directly involved in technology.

Answer (C) is incorrect because the trucking industry


has been deregulated.

Answer (D) is incorrect because the tax would be an


indirect method of economic intervention.

[24] Source: CMA 0691 1-30

Answer (D) is correct. In recent years, many


industries that were formerly highly regulated have
been deregulated to some extent. Airlines, banks,
railroads, and trucking have all been deregulated. The
steel industry has never been highly regulated. Thus,
there was no opportunity for steel to be deregulated.
[25] Source: CMA 1291 1-19

[27] Source: CMA 1291 1-21


Answer (A) is incorrect because all regulated
industries are not exempt from antitrust laws.
Answer (B) is incorrect because some regulated
industries are exempt or partially exempt from the
antitrust laws.

Answer (A) is incorrect because the Sherman Act


makes illegal every restraint of trade in interstate or
foreign commerce.

Answer (C) is incorrect because Congress, not the


applicable regulatory agency, determines which
industries are exempt from specific laws.

Answer (B) is incorrect because the Sherman Act


prohibits resale price maintenance.

Answer (D) is correct. Regulated industries such as


public utilities and banks may not be covered by the
U.S. antitrust laws. For example, the need for
monopoly status and tight regulation by state agencies
make energy utilities essentially exempt from antitrust
laws. Banks and insurance companies, however, are
less tightly regulated and are subject to some
provisions of the antitrust laws. Thus, different laws
apply to different regulated industries.

Answer (C) is correct. The various exemptions from


antitrust laws include intrastate commerce, labor
unions, regulated utilities, patents, copyrights, and
reasonable noncompetition clauses between buyers
and sellers of businesses, partners in a partnership,
and purchasers of technology or equipment.
Agricultural and fishing organizations, financial
institutions, broadcasting, transportation industries,
and professional baseball also have varying levels of
exemption. Moreover, in 1982, Congress enacted
the Export Trading Company Act. The purpose of
the ETCA is to encourage export of U.S. produced
goods and services. Persons who wish to form an
export trading company may obtain from the
Commerce Department a certificate of antitrust
immunity (called a certificate of review) after
concurrence by the Justice Department. This
certificate is issued if certain conditions are met.
These prohibit unfair export competition, substantial
lessening of competition or restraint of trade in the
U.S., unreasonable effects on U.S. prices, and sale
or resale of the goods or services in the U.S.
Answer (D) is incorrect because the Sherman Act
makes illegal every restraint of trade in interstate or
foreign commerce.
[26] Source: CMA 1291 1-20
Answer (A) is incorrect because pollution would not
be totally eliminated. Some firms might find that a low
level of pollution is more cost beneficial than total
elimination.
Answer (B) is correct. The setting of effluent
standards has often been criticized as an inefficient
method of pollution control. Economists prefer a

[28] Source: CMA 1291 1-22


Answer (A) is correct. A federal regulatory agency
may regulate some aspect of all industries or may
regulate a specific industry in accordance with power
delegated by Congress in the enabling legislation.
Agency functions include executive, adjudicatory, and
rulemaking activities. Such agencies, however, may
not impose taxes.
Answer (B) is incorrect because federal regulatory
agencies have the power to issue rules and
regulations.
Answer (C) is incorrect because federal regulatory
agencies have the power to investigate violations of
statutes and rules.
Answer (D) is incorrect because federal regulatory
agencies have the power to recommend penalties for
violations of statutes and rules.
[29] Source: CMA 1291 1-24
Answer (A) is correct. Title VII of the Civil Rights
Act of 1964 prohibits employment discrimination on
the basis of race, color, national origin, religion, or
sex. Under the Act, a pervasive pattern or practice of
discrimination, the employer's adoption of a neutral

rule having an adverse impact on a protected class,


and the adoption of a neutral rule that perpetuates
past discrimination are violations that often must be
proved, at least in part, by statistical evidence. A
controversial remedy sometimes adopted in such
cases is an affirmative action order, which provides
preferences to members of the class that previously
suffered from discrimination. Affirmative action
preferences apply even though the specific persons
benefited are not necessarily those who were
victimized by illegal discrimination. Affirmative action
programs are sometimes criticized because
employment preferences are often viewed as reverse
discrimination. Moreover, they may not result in the
hiring, retention, and promotion of the most
productive workers.
Answer (B) is incorrect because a work force in the
same proportion as the demographics of the total
population is not in conflict with the equal
employment opportunity laws.
Answer (C) is incorrect because even a bona fide
seniority system may violate Title VII if it is found to
perpetuate past discrimination. An affirmative action
remedy would necessarily have the effect of
weakening such a system.

Answer (B) is correct. Economic regulation usually


concerns price and service to the public and is
ordinarily industry specific. Social regulation has
broader objectives and more pervasive effects. It
addresses quality of life issues, such as workplace
and product safety, pollution, and fair employment
practices, and it applies to most industries. Social
regulation has been criticized on the grounds that it is
costly, contributes to overregulation, may inhibit
innovation, increases inflation and may place a
disproportionate burden on small companies, thereby
having an anticompetitive effect. Another criticism is
that regulators are perceived to have little concern for
the relation of marginal benefits and costs. For
example, achieving total compliance with the
standards for treated wastewater may be twice as
costly as attaining 95% of those standards. In other
words, the relatively small improvements sometimes
sought by regulators may be viewed as excessively
costly by those who are regulated.
Answer (C) is incorrect because OSHA and the
EPA are criticized for maintaining rigid standards that
are mandated regardless of cost.
Answer (D) is incorrect because compliance costs
are often quite high.

Answer (D) is incorrect because wages should be


directly related to marginal productivity.
[32] Source: CMA 1291 1-28
[30] Source: CMA 1291 1-25
Answer (A) is incorrect because the CPSC does not
levy fines, although it may bring suit for violations of
the Act, and a court may levy fines as a result.
Answer (B) is correct. The Consumer Product Safety
Act created the Consumer Product Safety
Commission, which promotes voluntary safety
standards, develops and enforces mandatory
standards, prohibits unsafe products if safety
standards will not be sufficient, recalls hazardous
products, furnishes information to consumers, and
works with state and local governments. Thus, if a
product is hazardous and presents an unreasonable
risk of injury and safety standards will not adequately
protect the public, the CPSC may issue an
appropriate order banning the product. If a hazard is
deemed to be imminent, it may seek a U.S. District
Court's authorization to seize the product or halt its
distribution. However, the emphasis of the
Commission's work is the prevention of problems
through standard setting, not the punishment of
wrongdoers.
Answer (C) is incorrect because consumer choice is
lessened when the CPSC keeps unsafe products off
the market.
Answer (D) is incorrect because the cost of
regulation results in higher costs for regulated
products.

Answer (A) is incorrect because the Act does not


require financial reports by management.
Answer (B) is incorrect because the Taft-Hartley Act
of 1947 designates a list of unfair labor practices on
the part of unions.
Answer (C) is correct. The National Labor Relations
Act, or Wagner Act, was designed to control the
supply of labor while guaranteeing workers the right
to organize, that is, to form, join, and assist labor
unions. It also prohibited certain unfair labor practices
by management. Moreover, it permits employees to
elect a collective bargaining agent by majority vote
and provides for the NLRB. The NLRB is a federal
agency that hears representation cases and
determines whether actions by labor or management
constitute unfair labor practices.
Answer (D) is incorrect because the Landrum-Griffin
Act of 1959 democratized unions.
[33] Source: CMA 1291 1-29
Answer (A) is incorrect because the Act did not
permit strikes for health and safety violations.
Answer (B) is incorrect because the union shop is still
allowed in many states; the closed shop was
outlawed.
Answer (C) is incorrect because the Act limited the
power of unions, not of employers.

[31] Source: CMA 1291 1-26


Answer (A) is incorrect because quality of life is the
primary objective of these laws.

Answer (D) is correct. After the NLRA was passed


in 1935, unions became very powerful, strikes were
crippling, and employees were coerced by labor

bosses. The Taft-Hartley Act placed restraints on


unions that resulted in a more even balance of power
between unions and employers. Among its many
provisions, the 1947 Act prohibited certain unfair
labor practices by unions: coercion of employees to
join unions, discrimination against nonunion
employees except when a valid union shop agreement
is in place, refusing to bargain in good faith,
secondary strikes, featherbedding (payment by
employers for work not performed), and charging
new members excessive initiation fees.
[34] Source: CMA 1291 1-30
Answer (A) is incorrect because comparable worth
does not address comparability of U.S. and foreign
wages.
Answer (B) is correct. The equal employment
opportunity laws require employees to receive equal
compensation for equal work. But compensation
based on comparable worth would require workers
to be paid for the value of their contributions to the
organization or to society, not on the basis of the
value established by market forces that may reflect
entrenched sexist attitudes. Thus, workers who
perform comparable but not equal work, that is,
different jobs needing different skills, would be
compensated equally. Comparable worth advocates
believe that the application of this principle would
remedy the underpayment of persons in occupations
historically dominated by women. The most severe
application of comparable worth law exists in the
Canadian Province of Ontario where it has invoked
much criticism because of the high costs relative to
the benefits derived. Some Ontario companies have
spent many thousands of dollars determining the
extent of a comparable w orth problem only to have
the result show that one or two female workers were
underpaid by a few hundred dollars.
Answer (C) is incorrect because comparable worth
does not address wage rates for the same job in
union and non-union environments.
Answer (D) is incorrect because comparable worth
does not address wage rates for the same job in the
government and the private sector.

adequate information. The Securities Acts of 1933


and 1934 were not a direct result of the stock market
crash; instead, it was the bankruptcy of Ivar Kreuger,
the greatest swindler the world has ever known, that
led to the passage of the securities acts.
[36] Source: CMA 0693 1-14
Answer (A) is incorrect because the purpose of a
private placement is to avoid SEC regulation.
Answer (B) is incorrect because, in the absence of
federal regulation, private placements typically take
less time than public offerings.
Answer (C) is correct. Private placements in
unlimited amounts are exempted from the Securities
Act of 1933. A private placement may be offered
only to knowledgeable and sophisticated investors,
may be offered to no more than 35 nonaccredited
purchasers, and may not be offered to the general
public. However, the securities are restricted, and the
issuer must exercise reasonable care in determining
that purchasers are not underwriters and are
purchasing solely for their own investment purposes.
Private placement flotation expenses may be lower
than with a public offering, but some flotation costs
will still be incurred.
Answer (D) is incorrect because a private placement
of securities can be tailor-made to the borrower's
needs.
[37] Source: CMA 0693 1-13
Answer (A) is incorrect because blue-sky laws are
state, not federal, financial regulatory laws.
Answer (B) is incorrect because blue-sky laws are
state laws and have nothing to do with goodwill.
Answer (C) is correct. Blue-sky laws are state laws
designed to prevent fraudulent or misleading security
issues. The name comes from some of the earliest
laws that prohibited "everything under the blue skies
that is fraudulent."
Answer (D) is incorrect because blue-sky laws apply
to all companies in a given state.

[35] Source: CMA 0693 1-11


Answer (A) is correct. The 1934 Act was designed
to regulate securities after initial issuances. Other
purposes of the 1934 Act were to provide adequate
information to investors and to prevent insiders from
unfairly using their information.
Answer (B) is incorrect because the Securities Act of
1933 was designed to regulate new issues of
securities.
Answer (C) is incorrect because the objective is not
to protect investors from losses, but to provide
investors with adequate information upon which to
base investment decisions.
Answer (D) is incorrect because the objective was to
regulate securities and provide investors with

[38] Source: CMA 0693 1-19


Answer (A) is incorrect because small businesses do
have a significant impact on jobs and the
environment; however, they often cannot absorb the
additional costs brought on by regulation.
Answer (B) is incorrect because Congress has
elected to exempt small businesses from some social
regulation because of the cost burden on such firms.
Answer (C) is correct. Social regulation embraces a
variety of government requirements related to matters
such as consumer protection, environmental quality,
employment discrimination, and workplace safety.
Small businesses have sometimes been exempted
from social regulations, because Congress has felt

that compliance costs would be an excessive burden


for small firms. These costs are relatively fixed and
are therefore higher per unit of output for smaller than
for larger firms. Compliance costs therefore create a
competitive disadvantage for small firms relative to
large firms.
Answer (D) is incorrect because the cost burden is a
more critical factor than small business lobbyists.
[39] Source: CMA 0693 1-20
Answer (A) is incorrect because the standards were
established without consideration for costs of
compliance.
Answer (B) is correct. Under the Clean Air Act,
individuals can initiate or participate in civil
enforcement actions. Individuals may also seek
damages for personal injuries suffered as a result of
violations.
Answer (C) is incorrect because national ambient air
quality standards (NAAQS) have been established to
prevent damage to items such as visibility, crops,
waterways, and buildings.
Answer (D) is incorrect because national ambient air
quality standards (NAAQS) have been established
with respect to specified pollutants to protect human
health and the public welfare.
[40] Source: CMA 0693 1-21
Answer (A) is incorrect because the CPSC is not
dependent upon manufacturers to provide information
on product safety. Much information is obtained from
consumers.
Answer (B) is incorrect because standards are
compulsory.
Answer (C) is incorrect because cost-benefit analysis
is not a consideration when products are dangerous.
Answer (D) is correct. A manufacturer, retailer, or
distributor must notify the CPSC if it has reason to
know that a product poses a "substantial product
hazard" because it violates a CPSC safety rule or for
other reasons. In these circumstances, the CPSC
may order the party to notify those affected, repair or
replace the product, or submit its own corrective
action plan.
[41] Source: CMA 0693 1-22
Answer (A) is incorrect because boycotts are often
used by consumer groups attempting to influence a
manufacturer's behavior.
Answer (B) is incorrect because shareholder
resolutions are sometimes used in an attempt to
influence the boards of large corporations.
Answer (C) is correct. Boycotts of a manufacturer's
products are the most common tactic used by
consumer groups to induce a company to change its

behavior or the nature of its advertising. Boycotts


have sometimes been quite successful. Shareholder
resolutions have also been made at stockholder
meetings, but such resolutions have rarely been
successful. Lobbying for new laws is also an often
successful tactic. Occasionally, cooperating with the
manufacturer to find common ground has been used.
Creating a competing company, however, is not a
consumerism tactic -- at least not yet.
Answer (D) is incorrect because cooperative
meetings are sometimes used by consumer groups.
[42] Source: CMA 0693 1-23
Answer (A) is incorrect because OSHA encourages
labor-management committees to formulate safety
and health programs.
Answer (B) is incorrect because inspections are
intended to determine whether OSHA standards are
being followed. These standards relate to workplace
safety and health.
Answer (C) is incorrect because OSHA has the
authority to levy civil monetary penalties of up to
$1,000 per violation and up to $10,000 for a repeat
offense.
Answer (D) is correct. OSHA is empowered to
conduct surprise inspections to determine whether
standards are being met. However, an employer may
demand that the inspector obtain a search warrant.
Thus, no precipitating event must occur prior to an
inspection.
[43] Source: CMA 0693 1-24
Answer (A) is incorrect because most regulated
industries, such as public utilities, airlines, and banks,
are very capital intensive.
Answer (B) is incorrect because uncontrolled
monopoly power may result in excessive prices, poor
quality, and inadequate output.
Answer (C) is incorrect because regulated industries
are often considered to be vital. Indeed, some are
deemed to be sufficiently vital to justify public
ownership or the equivalent (e.g., the postal service).
Answer (D) is correct. Regulated industries are
usually those in which natural monopolies exist.
Regulation is imposed to prevent the abuses that may
flourish in a noncompetitive environment and to
ensure that consumers receive adequate service at
fair prices. Rates are established to provide a fair and
reasonable return to investors, not consumers.
[44] Source: CMA 0693 1-26
Answer (A) is incorrect because the Justice
Department and the FTC can enforce antitrust laws.
Answer (B) is incorrect because the a state attorney
general and the FTC can enforce antitrust laws.

Answer (C) is incorrect because individual citizens


and the FTC can enforce antitrust laws.
Answer (D) is correct. Under the Sherman Act,
violations may be civil or criminal. Criminal penalties
include fines and imprisonment and enforcement is by
the Justice Department. Civil suits may also be
brought by private parties, who may recover treble
damages. The FTC also has specific authority to
enforce the Clayton and Robinson-Patman Acts and
implied authority to enforce the Sherman Act. State
law may also provide remedies for antitrust violations.
The U.S. Department of Commerce is not entitled to
bring suit for violation of antitrust laws.
[45] Source: CMA 0693 1-25
Answer (A) is correct. The Sherman Act specifies
that some type of arrangements between competitors
are to be considered unreasonable without inquiry.
These are called per se violations. These include:
price fixing (agreeing to any price), division of
markets (agreeing where to sell), group boycotts
(agreeing not to deal with another), and resale price
maintenance (limitations on buyer's resale price). The
most critical of these in terms of number of violations
prosecuted and dollars involved is price fixing.
Answer (B) is incorrect because allocation of
markets, although a per se violation, has rarely been a
problem.
Answer (C) is incorrect because boycotts involving
third parties are not nearly as common as price fixing
prosecutions.
Answer (D) is incorrect because tie-in sales are
prohibited by the Clayton Act of 1914.
[46] Source: CMA 0693 1-27
Answer (A) is incorrect because many cases have
been filed under the Sherman Act; however, the rule
of reason has limited the number of successful
prosecutions.
Answer (B) is incorrect because numerous
companies still engage in anti-competitive conduct.
Answer (C) is incorrect because intent is significant;
for example, a company that has a monopoly thrust
upon it is not in violation.
Answer (D) is correct. The Sherman Antitrust Act of
1890 makes illegal every contract, combination, or
conspiracy in restraint of trade in interstate or foreign
commerce. Successful prosecutions have been few
because the courts developed a rule of reason
stipulating that only unreasonable restraints of trade
are illegal. Thus, courts balance the anti-competitive
effects against the pro-competitive effects of a
restraint of trade. Unless a restraint is unreasonable
by its very nature (a per se violation), the rule of
reason applies.
[47] Source: CMA 0693 1-28

Answer (A) is incorrect because the ADA bans


employment discrimination against people with
disabilities.
Answer (B) is incorrect because the ADA provides
tax incentives for compliance costs.
Answer (C) is incorrect because the ADA bans
discrimination against people with disabilities in
transportation.
Answer (D) is correct. Title I prohibits employment
discrimination against qualified individuals with
disabilities with respect to job applications, hiring,
promotion, training, pay, and termination. The Act
also establishes requirements for employers and
providers of public accommodations, public
transportation, and telecommunications. Employers
must make "reasonable accommodations" for
individuals with disabilities that do not impose an
undue burden, for example, by modifying facilities
and work schedules, obtaining equipment, etc.
However, no federal funds are provided for
implementation of ADA by employers.
[48] Source: CMA 0693 1-29
Answer (A) is incorrect because quality standards
were established by the Environmental Protection
Agency.
Answer (B) is incorrect because fees have been
established for firms that emit pollutants.
Answer (C) is correct. Environmental laws have
established quality standards with respect to water,
land, and air. One approach has permitted violations,
but subjects violators to sliding charges depending
upon the quantity of pollutants released. Companies
that are below the standard for a particular pollutant
can sell their rights to other firms in the industry. In
addition, tax laws have provided incentives for
environmental improvements, for example a tax credit
for individuals who installed insulation. Similarly,
windmills are encouraged by the tax laws. Moving
facilities to offshore sites where governmental
standards are less stringent might be undertaken by
individual companies, but such action is not a
governmental device.
Answer (D) is incorrect because companies may buy
and sell their rights to emit pollutants.
[49] Source: CMA 0693 1-30
Answer (A) is incorrect because the FDA can extend
patent lives to allow for time lost during the regulatory
review.
Answer (B) is incorrect because the FDA considers
benefit-risk tradeoffs.
Answer (C) is incorrect because the FDA maintains
purity standards.
Answer (D) is correct. The FDA was created by the
Federal Food, Drug, and Cosmetics Act of 1938 to

help maintain the safety of drugs, food, cosmetics,


and medicinal products and devices. It regulates the
testing, distribution, and sale of drugs. Upon receiving
a new drug application, the FDA conducts hearings
and investigates the merits of the drug in a process
that may require years. Thus, an application must be
supported by substantial evidence that the drug will
have the asserted effects.
[50] Source: CMA 1293 1-1
Answer (A) is incorrect because the Sherman Act
also applies to domestic corporations.
Answer (B) is incorrect because the Taft-Hartley Act
of 1947 outlawed closed shops.
Answer (C) is incorrect because monopolies in
interstate or foreign commerce are illegal; monopolies
in intrastate commerce are not covered by the
Sherman act. Also, a company must show an
objective interest to monopolize. Monopoly power is
lawful if it is obtained by superior businesses acumen.
Answer (D) is correct. Section 1 of the Sherman
Antitrust Act of 1890 makes illegal every contract,
combination, or conspiracy in restraint of trade in
interstate or foreign commerce. Some types of
arrangements between competitors are found
unreasonable without inquiry. These are per se
violations and include price fixing, division of markets,
group boycotts, and resale price maintenance.
Section 2 prohibits the acts of monopolizing or
attempting to monopolize.
[51] Source: CMA 1293 1-2

safety, environmental degradation, and fair


employment practices. The abuses addressed are
those that are difficult for market forces to remedy.
For example, consumers may purchase products on
the basis of price and quality, but without regard to
the environmental impact of their production, and
unsafe working conditions may be tolerated by
individuals who have few opportunities for other
employment.
Answer (B) is incorrect because, although consumer
and environmental groups may occasionally exercise
some lobbying power, they are typically underfunded
and would have little impact on legislatures in the
absence of an obvious need for social regulation.
Answer (C) is incorrect because there is great
difficulty in measuring both the benefits and costs of
most social regulation.
Answer (D) is incorrect because no regulation is
desirable, but some regulation is necessary when
market forces are ineffective.
[53] Source: CMA 1293 1-5
Answer (A) is incorrect because the Clean Air Act
does not make pollution illegal; it provides
comprehensive regulation of air quality.
Answer (B) is incorrect because regulating the
amount of pollution a specific company can emit is
not efficient. That is why the emphasis is on the
application of air quality standards in the air quality
control regions in the states. Moreover, the states are
required to identify the major, not all, sources of
pollution.

Answer (A) is incorrect because the forces of supply


and demand do not determine prices; a regulatory
agency sets limits on the price a natural monopoly can
charge.

Answer (C) is incorrect because the Act does not


promote social awareness about the harm due to
pollution; in fact, it allows some pollution if a
company is willing to pay to pollute.

Answer (B) is incorrect because firms cannot easily


enter the utility industry. Great amounts of capital are
needed to establish a natural monopoly.

Answer (D) is correct. The Clean Air Act allows


companies to pollute legally if they have a permit to
do so. Firms can buy or sell these permits, and the
result is essentially a free market that enables a firm to
seek out a technology that is most cost effective. If
the price of permits is too high, a firm will buy new
equipment that will not pollute. If permits can be
acquired at a low cost, a company may decide to
continue producing with its old, high-pollution
equipment.

Answer (C) is incorrect because quality standards


are set by the same regulatory agency that controls
prices.
Answer (D) is correct. Regulated industries such as
gas and electric utilities, most of which are natural
monopolies, have price and quality controls
established by state or local governmental regulatory
bodies. Such industries usually have inelastic demand
curves, which means regulation can help control
prices. Public utilities are monopolies because having
more than one of each type of utility in a locality
would be inefficient. Rates are normally set to
provide owners with a reasonable return on their
investment.
[52] Source: CMA 1293 1-4
Answer (A) is correct. Social regulation concerns
quality of life issues, e.g., workplace and product

[54] Source: CMA 1293 1-6


Answer (A) is incorrect because a merger between
firms in different and unrelated markets is a
conglomerate merger.
Answer (B) is incorrect because a merger between
two or more firms at different stages of the
production process is a vertical merger.
Answer (C) is incorrect because a merger between a
producer and a supplier is a vertical merger.
Answer (D) is correct. A horizontal merger is one

between competitors in the same market. From the


viewpoint of the Justice Department, it is the most
closely scrutinized type of merger because it has the
greatest tendency to reduce competition.
[55] Source: CMA 1293 1-7
Answer (A) is incorrect because unions raise wages
for their members and indirectly reduce wages in the
nonunionized labor market.
Answer (B) is correct. Labor unions have typically
worked for higher wages and better working
conditions for their members. Through collective
bargaining and restrictions on membership, unions
increase the wage rates of members in comparison
with those earned by nonunionized workers. Because
of the greater cost of hiring unionized workers, the
amount of labor demanded by employers is reduced.
The workers that are unemployed as a result may
then seek employment in the nonunion labor market,
thereby increasing the supply and decreasing the
wages in that market.
Answer (C) is incorrect because unions raise wages
for their members and indirectly reduce wages in the
nonunionized labor market.
Answer (D) is incorrect because unions raise wages
for their members and indirectly reduce wages in the
nonunionized labor market.
[56] Source: CMA 1293 1-8
Answer (A) is correct. A closed shop is a workplace
in which union membership is required as a condition
of obtaining employment. The closed shop was
outlawed by the Labor-Management Relations Act
(Taft-Hartley Act) of 1947. In contrast with a closed
shop, a union shop is still allowed in some states. In a
union shop, an employee is required to join the union
after employment. Still other states have
right-to-work laws that allow employees to work at
any job without union membership.

management without submitting to the complexities of


corporate formation and operation.
Answer (C) is correct. The most common form of
business organization is the sole proprietorship. A
proprietorship is easily and inexpensively organized
because no formalities are required. Moreover, it
consists of one person (the proprietor). It is created
under common law whenever a person enters into
business and does not choose another form or
organization.
Answer (D) is incorrect because a corporation is a
legal entity with a legal existence separate from its
owners; the formation and existence of corporations
is entirely regulated by state statute.
[58] Source: CMA 1293 1-11
Answer (A) is incorrect because the elimination of all
pollution would probably be too costly for the
resulting benefits.
Answer (B) is incorrect because firms are more
supportive of cost-benefit regulatory policies.
Answer (C) is incorrect because granting tax relief to
those firms which pollute the least is not a
cost-benefit policy.
Answer (D) is correct. If environmental policies
weigh the costs and benefits of regulation, some
pollution will exist because the costs of eliminating all
pollution will likely outweigh the benefits.
Cost-benefit considerations are reflected in, for
example, the establishment of air quality standards
that do not attempt to eliminate the incidence of
specified pollutants and requirements for control
technology that, for the most part, do not mandate
use of devices that achieve the maximum pollution
reduction. The drawback of a cost-benefit approach
is the difficulty of quantifying benefits.
[59] Source: CMA 1293 1-12

Answer (B) is incorrect because, in a closed shop, all


employees must be a member of the union.

Answer (A) is incorrect because the Federal Reserve


System regulates banks, but does not insure deposits.

Answer (C) is incorrect because collective bargaining


is necessary in a closed shop. All workers are
members of the union.

Answer (B) is incorrect because the FSLIC was the


federal agency that formerly insured savings and loan
associations; the FSLIC went bankrupt as a result of
the savings and loan crisis of the 1980s.

Answer (D) is incorrect because only workers who


are union members have the right to work in a closed
shop.
[57] Source: CMA 1293 1-9
Answer (A) is incorrect because a partnership is
more complex than a proprietorship, but less difficult
to form than a corporation.
Answer (B) is incorrect because a limited partnership
can be created only pursuant to a statute. It is a form
of business organization unknown at common law. A
limited partnership permits investors to avoid
personal liability and the duties of day-to-day

Answer (C) is correct. Commercial bank deposits


are insured up to $100,000 per account by the
Federal Deposit Insurance Corporation (FDIC), a
federal agency. It is funded by premiums paid by
member banks.
Answer (D) is incorrect because the treasury is not
the primary insurer of bank deposits, although it may
ultimately be asked by Congress to provide funds if
the FDIC encounters financial difficulties.
[60] Source: Publisher
Answer (A) is incorrect because, under NEPA,

federal agencies must give environmental


considerations a weight equal to but not greater than
that afforded nonenvironmental concerns. NEPA
augments the existing powers of federal agencies to
deal with these environmental matters.
Answer (B) is correct. The provisions of NEPA
focus on federal governmental actions. Federal
agencies are specifically directed to incorporate an
analysis of environment consequences in their
decision-making processes. Actions of private
persons are affected by NEPA only when federal
involvement (approval, funding, etc.) is necessary
before such persons may act (e.g., federal approval
before drilling for oil in ocean waters within U.S.
jurisdiction). Otherwise, NEPA does not directly
concern activities of private persons.
Answer (C) is incorrect because, under NEPA,
federal agencies must give environmental
considerations a weight equal to but not greater than
that afforded nonenvironmental concerns. NEPA
augments the existing powers of federal agencies to
deal with these environmental matters.
Answer (D) is incorrect because, under NEPA,
federal agencies must give environmental
considerations a weight equal to but not greater than
that afforded nonenvironmental concerns. NEPA
augments the existing powers of federal agencies to
deal with these environmental matters.
[61] Source: Publisher
Answer (A) is incorrect because the CWA broadly
prohibits any discharges of pollutants into waters
subject to the jurisdiction of the United States by any
person, except in compliance with the Act. Under the
CWA, impairment of navigation is irrelevant.
Answer (B) is incorrect because to be subject to
federal jurisdiction, and thus the CWA, the waters in
which pollutants are discharged must be so-called
"navigable waters," which are defined as waters of
the United States (including the territorial waters).
This broad definition does not encompass all bodies
of water located within the bounds of the United
States.
Answer (C) is incorrect because the Rivers and
Harbors Act of the late 1800s was the first major
piece of federal legislation promulgated to protect
U.S. waterways. Until the passage of the CWA, the
Rivers and Harbors Act was also used to combat
pollutive discharges, although its original purpose was
to keep waterways clear from obstruct ions to
navigation.
Answer (D) is correct. The CWA (1972)
substantially amended the Federal Water Pollution
Control Act of 1948. It seeks to restore and maintain
the physical and biological integrity of the waters of
the United States. Its objectives are to render water
suitable for recreation and propagation of fish and
other wildlife and to eliminate discharges of
pollutants.
[62] Source: Publisher

Answer (A) is incorrect because profit percentages


are not set by antitrust laws other than to the extent
that price discrimination is prohibited.
Answer (B) is incorrect because firms may enter into
joint ventures.
Answer (C) is correct. Antitrust laws are designed to
promote more efficient allocation of resources,
greater choice for consumers, greater business
opportunities, fairness in economic behavior, and
avoidance of concentrated political power resulting
from economic power. Competition results in greater
output and lower prices than other market structures.
Answer (D) is incorrect because competition results
in greater output and lower prices than other market
structures.
[63] Source: Publisher
Answer (A) is incorrect because a pricing agreement
among competitors, even if aimed at lowering prices,
is illegal per se because it is assumed that such an
agreement over the long run will substantially lessen
competition.
Answer (B) is incorrect because pricing agreements
aimed at eliminating unfair competition and fixing
prices are illegal.
Answer (C) is incorrect because it is presumed that in
the long run even reasonable and fair agreements will
contribute to a lessening of competition.
Answer (D) is correct. No pricing agreements among
competitors are legal because the antitrust laws forbid
competitors to agree on the price of the products
they sell.
[64] Source: Publisher
Answer (A) is incorrect because many cases have
held that 60% or less of the market constitutes
monopolistic power. Hence, the firm need not hold
100% of the market.
Answer (B) is incorrect because the government
need not prove that the defendant controls a
particular percentage of the market to establish that
an illegal monopoly or attempt to monopolize has
occurred.
Answer (C) is correct. The purpose of antitrust laws
is to preserve and promote competition. The working
definition of monopoly in antitrust law is that the
defendant has the power to control prices or exclude
competition. Under the Sherman Act, formation of or
the attempt to form a monopoly is illegal.
Answer (D) is incorrect because the Sherman Act
prohibits monopolies and attempts to monopolize.
Thus, proof of contracts, combinations, and
conspiracies in restraint of trade is not always
necessary.

[65] Source: Publisher


Answer (A) is incorrect because the accounting
requirements apply only to publicly held, registered
companies under the 1934 act.
Answer (B) is incorrect because the accounting
requirements apply only to publicly held, registered
companies under the 1934 act.
Answer (C) is correct. The accounting requirements
of the FCPA apply to all companies required to
register and report under the Securities Exchange Act
of 1934. These companies must maintain books,
records, and accounts in reasonable detail that
accurately and fairly reflect transactions. The FCPA
also requires these companies to maintain a system of
internal accounting control that provides certain
reasonable assurances, including that corporate
assets are not used for bribes. If payoffs are made,
they must be reflected in the company's records.
Answer (D) is incorrect because the accounting
requirements apply only to publicly held, registered
companies under the 1934 act.
[66] Source: Publisher
Answer (A) is correct. The main purpose of the
Foreign Corrupt Practices Act of 1977 is to prevent
bribery by firms that do business in foreign countries.
A major ramification is that it requires all companies
that must register with the SEC under the Securities
Exchange Act of 1934 to maintain adequate
accounting records and a system of internal
accounting control.
Answer (B) is incorrect because, although some
international accounting standards have been
promulgated, they are incomplete and have not
gained widespread acceptance.
Answer (C) is incorrect because there are no
requirements for providing periodic reports on foreign
commerce or foreign political party affiliations.
Answer (D) is incorrect because U.S. auditing
standards apply to the conduct of audits by
accountants.

Violations of the FCPA are federal felonies. The


penalties are 5 years in prison or a $10,000 fine or
both for an officer, director, or shareholder who
helps make the bribe.
[68] Source: CIA 0594 IV-67
Answer (A) is incorrect because the easy and
inexpensive organization is one of the most appealing
characteristics of sole proprietorships.
Answer (B) is incorrect because sole proprietors
make all management decisions, and have the rights
to all profits as well as the right to sell the business.
Thus, the entrepreneur has freedom of action.
Answer (C) is incorrect because, as the sole owner
and profit-sharer, a sole proprietor has a strong
incentive to manage the business efficiently so as to
earn more profits.
Answer (D) is correct. The sole proprietor does not
enjoy the benefits of specialization. (S)he must
perform all management functions, including all
decisions relating to buying and selling, the acquisition
and maintenance of personnel, and the technical
aspects of production, advertising, and distribution.
[69] Source: CIA 0594 IV-66
Answer (A) is incorrect because a sole
proprietorship is owned by one person. The sole
proprietor is personally liable for all debts.
Answer (B) is incorrect because general partners
share profits and losses of the venture. Debts of a
partnership are ultimately the debts of the individual
general partners.
Answer (C) is correct. A shareholder owns a
property interest in the underlying net assets of the
corporation and is entitled to share in its profits. But,
unlike a sole proprietor or a general partner, the
shareholder is not subject to liability beyond his/her
investment.
Answer (D) is incorrect because monopoly is not a
legal form of business organization.
[70] Source: CIA 0589 IV-47

[67] Source: Publisher


Answer (A) is incorrect because this is not covered
by the provisions in the FCPA.
Answer (B) is incorrect because this is not covered
by the provisions in the FCPA.
Answer (C) is incorrect because all U.S. firms are
subject to the anti-bribery provisions.
Answer (D) is correct. The Foreign Corrupt
Practices Act (FCPA) prohibits any U.S. firm from
making bribes to foreign officials to influence official
acts. The businesses subject to the FCPA include
corporations, partnerships, limited partnerships,
business trusts, and unincorporated organizations.

Answer (A) is correct. Unlike a corporation, a


partnership is not treated as a separate legal entity for
purposes of liability. A corporate shareholder is
ordinarily liable only to the extent of the investment,
but a general partner usually has full personal liability
for the obligations of the partnership.
Answer (B) is incorrect because double taxation is a
disadvantage of the corporate but not the partnership
form of business organization. Corporate income is
taxed at the corporate level, and the individual
shareholders pay taxes on dividends. A partnership,
however, is not a taxable entity.
Answer (C) is incorrect because partnerships can be
created without formalities if two or more persons

agree to become associated as co-owners to carry


on a business for profit. The agreement need not even
be explicit. A corporation, however, must be formed
according to specific statutory requirements.
Answer (D) is incorrect because partnerships are not
subject to as many regulatory requirements as
corporations.
[71] Source: CIA 0593 IV-47
Answer (A) is incorrect because a limited partnership
is not used to limit withdrawals.
Answer (B) is incorrect because a limited partnership
is not used to limit the duration of the partnership.
Answer (C) is correct. A primary characteristic of
limited partnerships is that they must have both
general and limited partners. General partners have
unlimited liability for the obligations of the partnership
and are entitled to manage the business. A limited
partner's liability is ordinarily limited to the amount of
the capital contribution to the partnership. (S)he does
not have a right to participate in management. A
limited partner who nevertheless has a role in
management may become personally liable for
partnership obligations to third parties aware of that
role.
Answer (D) is incorrect because a limited partnership
is not used to the authority of general partners to
manage the business.

the balance of power between labor and


management. It outlawed the closed shop, which
required union membership as a condition of
obtaining employment. In addition, the Taft-Hartley
Act required unions to bargain in good faith. Also, the
President was given authority to obtain an injunction
against strikes for an 80-day cooling-off period if
national health, welfare, or security was threatened.
Answer (D) is incorrect because the Taft-Hartley Act
did not apply to federal employees.
[74] Source: CMA 0694 1-18
Answer (A) is correct. The Robinson-Patman Act
prohibits price discrimination with respect to both
buyers and sellers. Price discrimination is allowed if it
is to meet a competitor's price quote or if justified by
the quantity purchased.
Answer (B) is incorrect because cost increases
caused by government specification is not an
allowable exception to the Robinson-Patman Act.
Answer (C) is incorrect because the nature of a
market is not a factor in price discrimination under the
Robinson-Patman Act.
Answer (D) is incorrect because the Justice
Department does not give approval for price
discrimination.
[75] Source: CMA 1294 1-10

[72] Source: CMA 0694 1-16


Answer (A) is incorrect because the Clayton Act of
1914 makes price discrimination, tying contracts,
anti-competitive mergers, and interlocking
directorates illegal.
Answer (B) is incorrect because unfair competition
and deceptive business practices are prohibited by
the Federal Trade Commission Act of 1914.
Answer (C) is incorrect because the Clayton Act of
1914 allows the Justice Department to prevent
mergers before they occur.
Answer (D) is correct. The Sherman Act makes
illegal every contract, combination, or conspiracy in
restraint of trade in interstate or foreign commerce.
Section 2 of the act prohibits monopolization or the
attempt to monopolize.

Answer (A) is incorrect because the Sherman Act of


1890 did not mention tying sales or interlocking
directorates.
Answer (B) is correct. The Clayton Act of 1914
prohibits mergers or acquisitions of stock that may
lessen competition or tend to create a monopoly.
Mergers can be prevented by the Justice Department
before they occur. Also prohibited by the Clayton
Act are sales that prevent the buyer from dealing with
the seller's competitors, tying (or tie-in) sales,
exclusive dealing, price discrimination among different
buyers, and interlocking directorates.
Answer (C) is incorrect because the Antitrust
Improvements Act of 1976 amended the antitrust
laws in various ways.
Answer (D) is incorrect because the Federal Trade
Commission Act of 1914 created the Federal Trade
Commission to enforce the Sherman and Clayton
acts.

[73] Source: CMA 0694 1-17


Answer (A) is incorrect because the closed shop was
outlawed by the Taft-Hartley Act.

[76] Source: CMA 1294 1-11

Answer (B) is incorrect because the Taft-Hartley Act


did not require binding arbitration.

Answer (A) is incorrect because the Taft-Hartley Act


provides for an 80-day injunction against a work
stoppage that might imperil national security.

Answer (C) is correct. The Labor-Management


Relations Act of 1947, better known as the
Taft-Hartley Act, placed restraints on unions to even

Answer (B) is correct. The Wagner Act gave


employees the right to join unions, bargain collectively
with employers, and engage in concerted activities for

collective bargaining. In effect, labor unions were


exempted from antitrust laws. The National Labor
Relations Board (NLRB) was formed to administer
the Act.
Answer (C) is incorrect because the Taft-Hartley Act
of 1947 provided for restraints on unions to permit a
more even balance of power between labor and
management.
Answer (D) is incorrect because the Landrum-Griffin
Act of 1959 requires that union officials be elected on
a regular basis and that secret ballots be used. It also
requires that unions submit financial reports to the
government.
[77] Source: CMA 1294 1-12
Answer (A) is incorrect because the Wagner Act of
1935 prohibited employers from interfering with the
right of employees to form unions.
Answer (B) is incorrect because the Taft-Hartley Act
required unions to bargain in good faith.
Answer (C) is incorrect because the
Norris-LaGuardia Act of 1932 outlawed yellow-dog
contracts.
Answer (D) is correct. The Taft-Hartley Act placed
restraints on unions that resulted in a more even
balance of power between labor and management.
One provision prohibits unions from coercing
employees to become union members. Thus, the
closed shop was outlawed. The act also requires
unions to bargain in good faith and provides for
mediation of disputes. The President of the USA was
given authority to obtain an injunction against strikes
or lockouts for an 80-day cooling-off period if the
national health, welfare, or security is threatened.
Also, secondary strikes and featherbedding are
prohibited.
[78] Source: CMA 1294 1-13
Answer (A) is incorrect because the FTC was
created in 1914 by the Federal Trade Commission
Act.
Answer (B) is incorrect because penalties were
stated in the 1890 Act.
Answer (C) is incorrect because the Clayton Act of
1914 prohibits mergers that substantially lessen
competition.
Answer (D) is correct. The Sherman Act makes
illegal every contract, combination, or conspiracy in
restraint of trade in interstate or foreign commerce.
Some types of arrangements between competitors
are found unreasonable without inquiry. These are
called per se violations and include price fixing,
division of markets, group boycotts, and resale price
maintenance.
[79] Source: CMA 1294 1-14

Answer (A) is incorrect because social legislation is


criticized when standards are rigid.
Answer (B) is incorrect because compliance costs
are typically high.
Answer (C) is incorrect because the free market is
normally not relied upon; with certain exceptions,
compliance is mandatory.
Answer (D) is correct. Economic regulation usually
concerns price and service to the public and is
ordinarily industry specific. Social regulation has
broader objectives and more pervasive effects. It
addresses quality of life issues, such as workplace
and product safety, pollution, and fair employment
practices, and it applies to most industries. Social
regulation has been criticized on the grounds that it is
costly, contributes to overregulation, may inhibit
innovation, increases inflation and may place a
disproportionate burden on small companies, thereby
having an anticompetitive effect. Another criticism is
that regulators are perceived to have little concern for
the relation of marginal benefits and costs. For
example, achieving total compliance with the
standards for treated wastewater may be twice as
costly as attaining 95% of those standards. In other
words, the relatively small improvements sometimes
sought by regulators may be viewed as excessively
costly by those who are regulated.
[80] Source: CMA 1295 1-20
Answer (A) is incorrect because some regulated
industries are subject to antitrust laws.
Answer (B) is correct. With respect to antitrust laws,
regulated industries are covered unless specifically
exempted by statute or the courts. For example,
regulated public utilities are specifically exempted
because they are viewed as natural monopolies.
Answer (C) is incorrect because the Justice
Department must look to statute or the courts to
enforce antitrust laws.
Answer (D) is incorrect because some regulated
industries, such as utilities, are exempt from antitrust
laws.
[81] Source: CMA 1295 1-21
Answer (A) is correct. The purpose of social
regulation is to require behavior that is thought to be
beneficial for other than purely economic
considerations. However, social regulation has costs,
which lead to increased prices and inflation, reduced
innovation and consumer choice, and decreased
productivity. Social regulation is criticized because
the marginal benefits may not equal the marginal
costs.
Answer (B) is incorrect because enforcement policies
are rarely viewed as lenient by those to whom social
legislation is directed.
Answer (C) is incorrect because social legislation
may show a concern for the quality of life by

regulating the quality of products.


Answer (D) is incorrect because social regulation
often involves noncompliance penalties.
[82] Source: CMA 1295 1-22

tend to create a monopoly, (2) sales that prevent the


buyer from dealing with the seller's competitors, (3)
tie-in sales (requiring a buyer to take other products
in order to buy the first product), (4) price
discrimination, and (5) interlocking directorates
(directors serve on the boards of competing firms).
The Clayton Act does not address unfair and
deceptive business practices such as false advertising.

Answer (A) is incorrect because the act does not


apply to export sales.
[85] Source: CMA 1295 1-25
Answer (B) is incorrect because cost differences
related to delivery are a justification for charging
different prices.
Answer (C) is incorrect because the act applies only
to sales in interstate commerce.
Answer (D) is correct. The Robinson-Patman Act of
1936, an amendment to the Clayton Act, outlaws
price discrimination that would lead to restraint of
trade. Both buyer and seller can be found guilty of
price discrimination under the provisions of the
Robinson-Patman Act. Price differentiation between
customers is allowed if there is a difference in costs.
For instance, quantity discounts are permitted if it can
be shown that larger quantities can be shipped with a
cost savings. Charging competing wholesalers
different prices for similar goods would be a violation
of the act.
[83] Source: CMA 1295 1-23

Answer (A) is correct. The acquisition of a shoe


retailer by a shoe manufacturer is an example of
vertical integration. Vertical integration is typified by a
merger or acquisition involving companies that are in
the same industry but at different levels in the supply
chain. In other words, one of the companies supplies
inputs for the other.
Answer (B) is incorrect because a conglomerate is a
company made up of subsidiaries in unrelated
industries.
Answer (C) is incorrect because market extension
involves expanding into new market areas.
Answer (D) is incorrect because product extension
involves selling additional products in the same
markets.
[86] Source: CMA 1295 1-26

Answer (A) is incorrect because quantity discounts


are not prohibited by the Sherman Act.

Answer (A) is incorrect because blue sky laws are


state laws.

Answer (B) is incorrect because the Sherman Act


does not apply to labor unions.

Answer (B) is incorrect because blue sky laws are


state laws.

Answer (C) is incorrect because the Sherman Act


does not prohibit mergers; only those that could lead
to restraint of trade are outlawed.

Answer (C) is correct. Blue sky laws are state laws


designed to prevent fraudulent or misleading security
issues. The name came from the fact that some of the
earliest laws prohibited "everything under the blue
skies which is fraudulent."

Answer (D) is correct. The Sherman Act of 1890


makes illegal every contract, combination, or
conspiracy in restraint of trade in interstate or foreign
commerce. Some types of arrangements, called per
se violations, are considered unreasonable without
inquiry. These violations include price fixing, division
of markets, group boycotts, and resale price
maintenance. Agreeing to submit identical bids on a
government contract would be a form of price fixing,
and thus a per se violation.

Answer (D) is incorrect because blue sky laws do


not regulate the environment; they regulate sales of
investment securities.
[87] Source: CMA 1295 1-27
Answer (A) is incorrect because the Taft-Hartley Act
did not address internal affairs such as financial
records and reports.

[84] Source: CMA 1295 1-24


Answer (A) is incorrect because this action is
prohibited by the Clayton Act.
Answer (B) is incorrect because this action is
prohibited by the Clayton Act.
Answer (C) is incorrect because this circumstance is
prohibited by the Clayton Act.
Answer (D) is correct. The Clayton Act of 1914
prohibits (1) mergers that may lessen competition or

Answer (B) is incorrect because the Wagner Act of


1935 gave more power to unions.
Answer (C) is incorrect because the Securities
Exchange Act of 1934 did not address reports by
labor unions.
Answer (D) is correct. The Landrum-Griffin Act of
1959 (the Labor Management Reporting and
Disclosure Act) requires unions to maintain financial
records and submit reports to the federal
government. The intent of the act was to extend the

provisions of the National Labor Relations Act to the


internal affairs of unions to make the organizations
more democratic and give members more rights.
[88] Source: CMA 1295 1-28
Answer (A) is incorrect because labor unions are
specifically exempted from antitrust regulation.

Answer (A) is correct. Although unions have


probably been as aggressive as ever in recent years,
union membership has declined. This decline is
attributable to the fact that growth has occurred in the
service industries rather than in the manufacturing
industries that have been the traditional strongholds of
unions.

Answer (B) is incorrect because intrastate commerce


is specifically exempted from antitrust regulation.

Answer (B) is incorrect because this is not a


compelling reason for the decline in union
membership.

Answer (C) is incorrect because patents and


copyrights are specifically exempted from antitrust
regulation.

Answer (C) is incorrect because this is not a


compelling reason for the decline in union
membership.

Answer (D) is correct. Several types of entities and


contracts are exempt from antitrust regulation. These
include firms not operating in interstate commerce,
labor unions, regulated public utilities, patents and
copyrights, agricultural and fishing organizations,
financial institutions, transport industries, professional
baseball, and companies qualifying under the Export
Trading Company Act. The telecommunications
industry is not exempt.

Answer (D) is incorrect because this is not a


compelling reason for the decline in union
membership.
[92] Source: CMA 0685 1-35
Answer (A) is incorrect because more members of
minorities have joined unions as the barriers of
discrimination have been removed and as their overall
representation in the labor force has increased.

[89] Source: CMA 1295 1-29


Answer (A) is incorrect because they are examples
of federal agencies enforcing social regulation.
Answer (B) is incorrect because they are examples of
federal agencies enforcing social regulation.
Answer (C) is incorrect because they are examples
of federal agencies enforcing social regulation.
Answer (D) is correct. Social regulation is intended
to benefit society as a whole. The Food and Drug
Administration (FDA), National Highway Traffic
Safety Administration, Consumer Product Safety
Commission, Occupational Safety and Health
Administration (OSHA), Environmental Protection
Agency (EPA), and Equal Employment Opportunity
Commission (EEOC) are all examples of social
agencies.
[90] Source: CMA 1295 1-30
Answer (A) is incorrect because the FTC is not
concerned with the regulation of foreign trade.

Answer (B) is incorrect because women have joined


unions as the barriers of discrimination have been
removed and as their overall representation in the
labor force has increased.
Answer (C) is correct. Because the growth areas of
the economy have been in the service industries, there
has been a decline in union membership among
blue-collar workers, who are largely in manufacturing
and maintenance occupations.
Answer (D) is incorrect because public sector
employment continues to grow while manufacturing
sector employment declines.
[93] Source: Publisher
Answer (A) is incorrect because a perfectly
competitive market was envisioned by classical
economics. Modern monopolies and oligopolies often
must be subject to regulation to enforce their social
responsibilities since the limits imposed by the market
are ineffective.

Answer (B) is incorrect because the FTC is not


concerned with the regulation of foreign trade.

Answer (B) is incorrect because the concept


embraces the public or societal interest.

Answer (C) is correct. The FTC Act of 1914


prohibits unfair methods of competition and unfair or
deceptive acts in commerce. The basic objectives are
to initiate antitrust actions and protect the consumer
public.

Answer (C) is correct. The concept of corporate


social responsibility involves more than serving the
interests of the organization and its shareholders.
Rather, it is an extension of responsibility to embrace
service to the public interest in such matters as
environmental protection, employee safety, civil
rights, and community involvement.

Answer (D) is incorrect because price discrimination


and unfair trade practices are merely elements of the
overall mission of the FTC.
[91] Source: CMA 0685 1-34

Answer (D) is incorrect because the concept


embraces the public or societal interest.

[94] Source: Publisher


Answer (A) is incorrect because such behavior may
prevent governmental action.
Answer (B) is incorrect because it is an argument for
such behavior.
Answer (C) is incorrect because it is an argument for
such behavior.
Answer (D) is correct. Socially responsible behavior
clearly has immediate costs to the entity, for example,
the expenses incurred in affirmative action programs,
pollution control, and improvements in worker safety.
When one firm incurs such costs and its competitor
does not, the other may be able to sell its products or
services more cheaply and increase its market share
at the expense of the socially responsible firm. The
rebuttal argument is that in the long run the socially
responsible company may maximize profits by
creating goodwill and avoiding or anticipating
governmental regulation.
[95] Source: CMA 0696 1-18
Answer (A) is incorrect because horizontal price
fixing occurs between two competitors at the same
level in the distribution process (e.g., between two
retailers in the same industry).
Answer (B) is correct. Resale price maintenance
agreements are a form of vertical price fixing because
a manufacturer or wholesaler restricts the price that
may be charged by a retailer. Such agreements are
per se violations of the Sherman Act of 1890.
Answer (C) is incorrect because preemptive buying is
not associated with resale price maintenance.
Answer (D) is incorrect because tying arrangements
require a buyer to purchase other products in
addition to the one that is desired.

[97] Source: CMA 0696 1-22


Answer (A) is incorrect because the closed shop was
prohibited by the Taft-Hartley Act of 1947.
Answer (B) is correct. The Clayton Act of 1914
prohibits price discrimination if it directly or indirectly
lessens competition such that it tends to create a
monopoly. The Robinson-Patman Act amended the
Clayton Act to prohibit sellers of goods from
granting, and buyers from inducing, unfair discounts
and other preferences. However, price differentials
are justified by a cost savings to the seller or a good
faith effort to meet, but not undercut, the lawful price
of a competitor.
Answer (C) is incorrect because group boycotts
were prohibited by the Sherman Act of 1890.
Answer (D) is incorrect because oligopolies are not
prohibited by the Clayton Act.
[98] Source: CMA 0696 1-23
Answer (A) is incorrect because interlocking
directorates are prohibited by the Clayton Act.
Answer (B) is incorrect because price discrimination
is prohibited by the Clayton Act as amended by the
Robinson-Patman Act.
Answer (C) is incorrect because tying contracts are
prohibited by the Clayton Act.
Answer (D) is correct. The Federal Trade
Commission Act of 1914, as amended by the
Wheeler-Lea Amendment of 1938, prohibits unfair
methods of competition and unfair or deceptive acts
in commerce, including false advertising. The act also
created the Federal Trade Commission (FTC) to
enforce the requirements of the act. The basic
objectives of the FTC are to initiate antitrust actions
and to protect the consumer public.

[96] Source: CMA 0696 1-21


[99] Source: CMA 0696 1-24
Answer (A) is incorrect because the SEC does not
have to approve a trust indenture.
Answer (B) is incorrect because the 1933 act
requires disclosure of nonexempted new issuances of
securities, including those of public utility holding
companies, not registration of particular entities.
Answer (C) is correct. The Securities Act of 1933
was designed to provide complete and fair disclosure
to potential investors. The 1933 act applies only to
the initial issuance of securities. Disclosure is
accomplished through the requirement that a
registration statement be filed with the SEC. Once
potential investors have complete disclosure, the
assumption is that they can make a reasonable
decision.
Answer (D) is incorrect because the Securities
Exchange Act of 1934 requires registration of
brokers.

Answer (A) is correct. The CPSC publishes product


safety standards, issues rules banning certain
hazardous products, brings federal suits to eliminate
dangers presented by imminently hazardous
consumer products, and requires manufacturers,
distributors, and retailers to give notice if they have
reason to know that their products present a
substantial hazard. Remedies include injunctive relief,
seizure of products, and civil and criminal penalties.
Answer (B) is incorrect because the CPSC must file
suit in federal court to have a product banned or
seized; it cannot ban or seize a product without a
court order.
Answer (C) is incorrect because the CPSC must file
suit in federal court to have a product banned or
seized; it cannot ban or seize a product without a
court order.

Answer (D) is incorrect because the CPSC may go


beyond asking for voluntary action.

[103] Source: CMA 0696 1-28


Answer (A) is incorrect because constitutional law is

[100] Source: CMA 0696 1-25


the fundamental law of a jurisdiction.
Answer (A) is incorrect because age may not be
asked under the provisions of the Age Discrimination
in Employment Act.
Answer (B) is incorrect because questions about a
disability may not be asked under the provisions of
the Americans with Disabilities Act of 1990.
Answer (C) is incorrect because marital status is not
a suitable subject for questioning under the civil rights
laws.
Answer (D) is correct. The Equal Employment
Opportunity Commission (EEOC) specifies several
questions that cannot be asked of applicants during
an employment interview. These include questions
regarding age, disability, arrest record, and marital
status. It is permissible to ask an applicant for
references.
[101] Source: CMA 0696 1-26
Answer (A) is incorrect because a closed shop
requires an employee to be a member of the union in
order to get a job.
Answer (B) is incorrect because a union shop
requires an employee to join the union after
employment.
Answer (C) is correct. Some states have
right-to-work laws that allow employees to work at
any job without union membership. Thus, employees
can decide not to be involved with a union even
though the union represents the workers in the shop.
Answer (D) is incorrect because paying dues is
equivalent to the requirements of a union shop.
[102] Source: CMA 0696 1-27
Answer (A) is incorrect because no labeling
requirements exist in an unregulated market.
Answer (B) is correct. Mandatory labeling follows
the disclosure approach. The basis of this approach is
the belief that regulation is sufficient if consumers have
the information needed to make rational decisions.
The assumptions are that consumers are sophisticated
enough to use the information provided and that
sellers will have an incentive to improve their
products.
Answer (C) is incorrect because stricter liability rules
promote product quality and safety, whereas labeling
requirements are primarily informational.
Answer (D) is incorrect because labeling
requirements do not create marketable property
rights. An example of such a regulatory practice is the
external trading of pollution rights.

Answer (B) is incorrect because statutory law is a


body of detailed enactments by the legislative branch.
Answer (C) is correct. Administrative law is
promulgated by the executive branch under a general
grant of authority to an agency to regulate an industry.
Administrative law may also be promulgated under a
specific grant of authority to an agency to make
detailed rules to achieve the objectives of a statute.
For example, the IRS makes rules to carry out
specific statutes but is not given the general authority
to make rules for the collection of revenue.
Administrative law may not go beyond the scope of
the statutes under which it is promulgated.
Answer (D) is incorrect because judicial (common)
law is created by the courts through the adjudication
of cases and the publication of the resulting opinions.
[104] Source: CMA 1296 1-20
Answer (A) is incorrect because the
Robinson-Patman Act concerns price discrimination,
not exclusive-dealing contracts.
Answer (B) is incorrect because exclusive-dealing
contracts are the subject of federal antitrust law. They
are not illegal under common law.
Answer (C) is incorrect because there is no
requirement to seek approval from the FTC.
Answer (D) is correct. The Clayton Act of 1914
prohibits exclusive-dealing requirements. However, a
franchise relationship is contractual, and the
agreement is in force for a specified period. Because
the franchisee and the franchisor have a common
public identity, an exclusive-dealing contract is
allowable if it is necessary to assure product quality.
Without the exclusive-dealing requirement, a
low-quality product sold by the franchisee could
reflect badly on the reputation of the franchisor.
[105] Source: CMA 1296 1-21
Answer (A) is correct. The basic purpose of the
federal securities laws in the United States, primarily
the Securities Act of 1933 and the Securities
Exchange Act of 1934, is to provide complete and
fair disclosure to potential investors. The emphasis is
on disclosure that allows informed investors to make
intelligent decisions.
Answer (B) is incorrect because the SEC does not
evaluate the merits of investments or define an
investment-grade security.
Answer (C) is incorrect because a corporation's
by-laws provide for voting rights. For example,
preferred stock rarely has a vote.

Answer (D) is incorrect because the federal laws


apply in all states regardless of the existence of
applicable state laws.
[106] Source: CMA 1296 1-22
Answer (A) is incorrect because the FTC was
established by the Federal Trade Commission Act of
1914.
Answer (B) is correct. The Sherman Act of 1890
makes illegal every contract, combination, or
conspiracy in restraint of trade in interstate or foreign
commerce. Some types of arrangements between
competitors are to be considered unreasonable
without inquiry. These are known as per se violations.
Price fixing, division of markets, group boycotts, and
resale price maintenance are per se violations.
Answer (C) is incorrect because price discrimination
is prohibited by the Robinson-Patman Act of 1936,
which was an amendment to the Clayton Act of
1914.
Answer (D) is incorrect because patent laws were in
effect long before the Sherman Act of 1890. Thus,
the U.S. Constitution (Article I, Section 8) grants to
Congress the power "To promote the progress of
science and useful arts, by securing for limited times
to authors and inventors the exclusive right to their
respective writings and discoveries."
[107] Source: CMA 1296 1-23
Answer (A) is incorrect because the Clayton Act of
1914 prohibits interlocking directorates in competing
companies.
Answer (B) is incorrect because the
Robinson-Patman Act of 1936 prohibits price
discrimination.
Answer (C) is correct. The Antitrust Improvements
Act of 1976 requires corporations with annual sales
or assets exceeding $100,000,000 to give advance
notice to the Justice Department and the FTC of any
acquisition of a corporation with annual sales or
assets of $10,000,000 or more.
Answer (D) is incorrect because the Sherman Act of
1890 prohibits restraint of trade and monopoly.

specific laws. Otherwise, the legislation or agency


terminates at the end of a specified period of time.
For example, the first federal Cost Accounting
Standards Board was established with a 10-year life.
At the end of that period, the agency was terminated
because Congress felt that its purpose had been
accomplished.
[109] Source: CMA 1296 1-25
Answer (A) is incorrect because shareholders in
publicly traded U. S. corporations have the right to
vote on fundamental corporate changes, to receive
declared dividends and annual reports, to vote, to
exercise any preemptive right that may have been
granted, to attend meetings, to inspect corporate
records, and to bring shareholder suits.
Answer (B) is incorrect because shareholders in
publicly traded U. S. corporations have the right to
vote on fundamental corporate changes, to receive
declared dividends and annual reports, to vote, to
exercise any preemptive right that may have been
granted, to attend meetings, to inspect corporate
records, and to bring shareholder suits.
Answer (C) is incorrect because shareholders in
publicly traded U. S. corporations have the right to
vote on fundamental corporate changes, to receive
declared dividends and annual reports, to vote, to
exercise any preemptive right that may have been
granted, to attend meetings, to inspect corporate
records, and to bring shareholder suits.
Answer (D) is correct. A corporation is owned by
shareholders who elect a board of directors to
manage the company. The board of directors then
hires managers to supervise operations. Shareholders
do not vote on major management changes because
the powers of the board include selection and
removal of officers and the setting of management
compensation. Shareholders do have the right to vote
on fundamental corporate changes, e.g., mergers and
acquisitions, any changes in the corporate charter and
bylaws, and dissolution.
[110] Source: CMA 1296 1-29
Answer (A) is incorrect because the ADA prohibits
employers from inquiring about a job applicant's prior
health insurance claims.
Answer (B) is incorrect because the ADA applies to
both physical and mental impairments.

[108] Source: CMA 1296 1-24


Answer (A) is incorrect because the Age
Discrimination in Employment Act provides
protection for older workers.
Answer (B) is incorrect because sunset laws are not
concerned with securities regulation.
Answer (C) is incorrect because laws that prohibit
the sale of alcohol on Sundays are blue laws.
Answer (D) is correct. Sunset laws are provisions
that require periodic review and reenactment of

Answer (C) is correct. The ADA requires


organizations with 25 or more employees to provide
reasonable accommodation for employees and job
applicants with disabilities. The ADA bans
employment discrimination against people with mental
or physical disabilities, provides tax incentives for
compliance costs, and requires remodeling of facilities
to provide access by individuals with disabilities.
Another provision of the ADA is that employers are
prohibited from inquiring into a job applicant's
disability with questions concerning medical history,
prior workers' compensation or health insurance

claims, work absenteeism due to illness, past


treatment for alcoholism, or mental illness.
Answer (D) is incorrect because the ADA provides
no federal funds for implementation of its provisions.
[111] Source: CMA 1296 1-30
Answer (A) is incorrect because monetary penalties
were imposed for insider trading before 1988. For
example, the Securities Exchange Act of 1934
requires insiders to turn over to the corporation any
short-swing profits on purchases and sales of
corporate stock.
Answer (B) is correct. The Insider Trading Sanctions
Act of 1984 extended the law to include those who
aid and abet insider trading. It also imposed a
treble-damages penalty. The 1988 act increased
prison terms from 5 to 10 years, the maximum fine
for individuals to $1,000,000, and the maximum fine
for corporations to $2,500,000. In addition, the SEC
now has authority to reward an informant with up to
10% of the fine levied against the perpetrator.
Answer (C) is incorrect because, under section 10(b)
of the 1934 act and Rule 10b-5, insider trading is a
fraudulent act subject to criminal penalties.
Answer (D) is incorrect because the SEC already
held the right to recover gains resulting from the illegal
use of insider information.
[112] Source: CMA 0697 1-22
Answer (A) is correct. SEC Rule 415 allows
corporations to file registration statements covering a
stipulated amount of securities that may be issued
over the 2-year effective period of the statement. The
securities are placed on the shelf and issued at an
opportune moment without the necessity of filing a
new registration statement, observing a 20-day
waiting period, or preparing a new prospectus. The
issuer is only required to provide updating
amendments or to refer investors to quarterly and
annual statements filed with the SEC. Shelf
registration is most advantageous to large
corporations that frequently offer securities to the
public.
Answer (B) is incorrect because an indenture is a
contract associated with the issuance of a debt
instrument.
Answer (C) is incorrect because a secondary market
registration does not expedite the issuance of
securities.
Answer (D) is incorrect because a red-herring is a
preliminary prospectus issued during the 20-day
waiting period before the related registration
statement becomes effective.

Act of 1950 prohibits the acquisition of the stock or


assets of another business if the effect may be to
lessen competition substantially or to create a
monopoly.
Answer (B) is incorrect because the Federal Trade
Commission Act of 1914 prohibits unfair methods of
competition in or affecting interstate commerce. It
also created the Federal Trade Commission, which
has authority to enforce the Clayton and
Robinson-Patman Acts. The FTC also is authorized
to proceed against unfair or deceptive acts or
practices.
Answer (C) is incorrect because the Sherman Act of
1890 makes illegal every contract, combination, or
conspiracy that unreasonably restrains trade in
interstate or foreign commerce. It also prohibits
monopolization and attempts and conspiracies to
monopolize.
Answer (D) is correct. The Robinson-Patman Act of
1936 amended the Clayton Act with respect to price
discrimination. Price discrimination by both buyers
and sellers is prohibited in interstate commerce of
goods of like grade and quality. The purpose of the
act is to protect competition. However, price
differentials are allowed if justified by a cost savings
to the seller or a good-faith effort to meet a
competitor's lawful price.
[114] Source: CMA 0697 1-26
Answer (A) is incorrect because the Federal Trade
Commission Act of 1914 prohibits unfair methods of
competition in or affecting interstate commerce. It
also created the Federal Trade Commission, which
has authority to enforce the Clayton and
Robinson-Patman Acts. The FTC also is authorized
to proceed against unfair or deceptive acts or
practices.
Answer (B) is correct. The Securities Exchange Act
of 1934 addresses the issue of insider trading.
Specifically, insiders must turn over to the
corporation any profits earned on purchases and
sales of their company's stock that fall within six
months of each other. They are also prohibited from
buying or selling stock based on inside information
not available to the public.
Answer (C) is incorrect because the Clayton Act of
1914 was intended to prevent monopolies. A
probability of a significant anticompetitive effect is a
basis for most violations of the act. It specifically
addresses price discrimination, tying contracts,
exclusive dealing arrangements, mergers, and
interlocking directorates.
Answer (D) is incorrect because the North American
Free Trade Agreement is an agreement providing for
free trade among the USA, Canada, and Mexico.
[115] Source: CMA 0697 1-27

[113] Source: CMA 0697 1-25


Answer (A) is incorrect because the Celler-Kefauver

Answer (A) is incorrect because on-time departure


standards are an aspect of the airline industry that has

been deregulated.
Answer (B) is incorrect because domestic fare
schedules are an aspect of the airline industry that has
been deregulated.
Answer (C) is correct. Airline operations have been
almost fully deregulated by the U. S. government, for
example, with regard to departure standards, fare
schedules, and the opportunity to eliminate
unprofitable routes. Despite deregulation, however,
airlines are still subject to federal and state labor
laws, including those governing collective bargaining.
The National Labor Relations Board exercises its
jurisdiction over all transportation enterprises that
furnish interstate services.
Answer (D) is incorrect because international fare
schedules are an aspect of the airline industry that has
been deregulated.
[116] Source: CMA 0697 1-28
Answer (A) is correct. Federal administrative
agencies belong to the executive branch of
government. They have quasi-legislative powers
(rulemaking authority pursuant to enabling statutes
enacted by Congress), quasi-judicial powers (the
ability to adjudicate certain disputes within their
jurisdiction), and quasi-executive powers (for
example, to investigate and prosecute violations).
However, no federal agency has taxing authority, a
power reserved to Congress. Moreover, all bills for
raising revenue must originate in the House of
Representatives.
Answer (B) is incorrect because it is a frequently
heard criticism of federal regulatory agencies and
policies.
Answer (C) is incorrect because it is a frequently
heard criticism of federal regulatory agencies and
policies.
Answer (D) is incorrect because it is a frequently
heard criticism of federal regulatory agencies and
policies.
[117] Source: CMA 0697 1-29
Answer (A) is incorrect because insider trading is
prohibited by Section 10(b) of the Securities
Exchange Act of 1934 and by the SEC's Rule 10b-5.

is essentially a disclosure statute. The SEC does not


evaluate the merits of securities. Its role is to enforce
the laws ensuring the public availability of information
to potential investors.
[118] Source: Publisher
Answer (A) is incorrect because the definition of the
term "hazardous substance" as used in CERCLA
does not include petroleum or any derivatives thereof
or natural gas.
Answer (B) is incorrect because the definition of the
term "hazardous substance" as used in CERCLA
does not include petroleum or any derivatives thereof
or natural gas.
Answer (C) is correct. Asbestos is a hazardous
substance subject to the provisions of CERCLA.
Answer (D) is incorrect because the definition of the
term "hazardous substance" as used in CERCLA
does not include petroleum or any derivatives thereof
or natural gas.
[119] Source: Publisher
Answer (A) is incorrect because under NEPA,
federal agencies must give environmental
considerations a weight equal to, but not greater than,
that afforded nonenvironmental concerns.
Answer (B) is correct. The provisions of NEPA
focus on federal governmental actions. Federal
agencies are specifically directed to incorporate an
analysis of environmental consequences in their
decision-making processes. Actions of private
persons are affected by NEPA only when federal
involvement (approval, funding, etc.) is necessary
before such persons may act (e.g., federal approval
before drilling for oil in ocean waters within U.S.
jurisdiction). Otherwise, NEPA does not directly
concern activities of private persons.
Answer (C) is incorrect because under NEPA,
federal agencies must give environmental
considerations a weight equal to, but not greater than,
that afforded nonenvironmental concerns.
Answer (D) is incorrect because NEPA augments the
existing powers of federal agencies to deal with these
environmental matters.
[120] Source: Publisher

Answer (B) is incorrect because an issuer that wishes


to make an interstate offering of new securities to the
public must file a registration statement with the SEC.
Answer (C) is incorrect because the Securities
Exchange Act of 1934 provides rules with regard to
proxy solicitations and tender offers.
Answer (D) is correct. The SEC is charged with
enforcement of federal securities laws. Under the
securities Act of 1933, the offer or sale of a security
to the public requires registration with the SEC
absent a specific exemption. However, the 1933 act

Answer (A) is incorrect because the CWA broadly


prohibits any discharges of pollutants into waters,
except if in compliance with the act. Impairment of
navigation is irrelevant.
Answer (B) is incorrect because, to be subject to the
CWA, the waters must be "navigable waters."
Answer (C) is incorrect because the Rivers and
Harbors Act of the late 1800s was used to combat
pollutive discharges, although its original purpose was
to keep waterways clear from obstructions to

navigation.

rights, and community involvement.

Answer (D) is correct. The CWA (1972)


substantially amended the Federal Water Pollution
Control Act of 1948. It seeks to restore and maintain
the physical and biological integrity of the waters of
the United States. Its objectives are to render water
suitable for recreation and propagation of fish and
other wildlife and to eliminate discharges of
pollutants.

Answer (D) is incorrect because the concept


embraces the public or societal interest.

[121] Source: CMA 1294 1-9


Answer (A) is incorrect because no information is
given regarding costs. Only selling prices and
quantities are given.
Answer (B) is incorrect because AC represents
greater satisfied demand.
Answer (C) is correct. The area ABC represents the
efficiency gain from antitrust action. It is the increase
in productivity (benefits to society) from the increased
quantity and lower prices resulting from the antitrust
action.
Answer (D) is incorrect because total benefit is
represented by the greater quantity produced (area
CBQ2Q1).

[124] Source: Publisher


Answer (A) is incorrect because such behavior may
prevent governmental action.
Answer (B) is incorrect because each is an argument
for such behavior.
Answer (C) is incorrect because each is an argument
for such behavior.
Answer (D) is correct. Socially responsible behavior
clearly has immediate costs to the entity, for example,
the expenses incurred in affirmative action programs,
pollution control, and improvements in worker safety.
When one firm incurs such costs and its competitor
does not, the other may be able to sell its products or
services more cheaply and increase its market share
at the expense of the socially responsible firm. The
rebuttal argument is that in the long run the socially
responsible company may maximize profits by
creating goodwill and avoiding or anticipating
governmental regulation.
[125] Source: Publisher

[122] Source: Publisher


Answer (A) is incorrect because "practitioners of
management accounting and financial management
have an obligation to the public, their profession, the
organization they serve, and themselves, to maintain
the highest standards of ethical conduct."
Answer (B) is incorrect because the audit committee
would be consulted first only if it were the next higher
managerial level.
Answer (C) is correct. To resolve an ethical problem,
the financial manager/management accountant's first
step is usually to consult his/her immediate superior. If
that individual is involved, the matter should be taken
to the next higher level of management.

Answer (A) is incorrect because it states an aspect of


the competence requirement.
Answer (B) is correct. According to the IMA Code
of Ethics, financial managers/management
accountants must "avoid actual or apparent conflicts
of interest and advise all appropriate parties of any
potential conflict."
Answer (C) is incorrect because it states an aspect of
the confidentiality requirement.
Answer (D) is incorrect because it states an aspect of
the competence requirement.
[126] Source: Publisher

Answer (D) is incorrect because if the superior is


involved, the next higher managerial level should be
consulted first.
[123] Source: Publisher
Answer (A) is incorrect because a perfectly
competitive market was envisioned by classical
economics.
Answer (B) is incorrect because the concept
embraces the public or societal interest.
Answer (C) is correct. The concept of corporate
social responsibility involves more than serving the
interests of the organization and its shareholders.
Rather, it is an extension of responsibility to embrace
service to the public interest in such matters as
environmental protection, employee safety, civil

Answer (A) is incorrect because the code does not


address these matters.
Answer (B) is incorrect because the code does not
address these matters.
Answer (C) is correct. Financial
managers/management accountants may not dis close
confidential information acquired in the course of their
work unless authorized or legally obligated to do so.
They must inform subordinates about the
confidentiality of information and monitor their
activities to maintain that confidentiality. Moreover,
financial managers/management accountants should
avoid even the appearance of using confidential
information to their unethical or illegal advantage.
Answer (D) is incorrect because other employment
may be accepted unless it constitutes a conflict of

interest.
[127] Source: CMA 1
Answer (A) is incorrect because the competence
standard pertains to the financial
manager/management accountant's responsibility to
maintain his/her professional skills and knowledge. It
also pertains to the performance of activities in a
professional manner.
Answer (B) is incorrect because the confidentiality
standard concerns the financial manager/management
accountant's responsibility not to disclose or use the
firm's confidential information.
Answer (C) is correct. One of the responsibilities of
the financial manager/management accountant under
the integrity standard is to "recognize and
communicate professional limitations or other
constraints that would preclude responsible judgment
or successful performance of an activity."
Answer (D) is incorrect because objectivity is the
fourth part of the IMA Code of Ethics. It requires
that information be communicated "fairly and
objectively," and that all information that could
reasonably influence users be fully disclosed.
[128] Source: CMA 2
Answer (A) is incorrect because the competence
standard pertains to the financial
manager/management accountant's responsibility to
maintain his/her professional skills and knowledge. It
also pertains to the performance of activities in a
professional manner.
Answer (B) is incorrect because the confidentiality
standard concerns the financial manager/management
accountant's responsibility not to disclose or use the
firm's confidential information.
Answer (C) is correct. The integrity standard requires
the financial manager/management accountant to
"refuse any gift, favor, or hospitality that would
influence or would appear to influence his/her actions.
Answer (D) is incorrect because objectivity is the
fourth part of the IMA Code of Ethics. It requires
that information be communicated "fairly and
objectively," and that all information that could
reasonably influence users be fully disclosed.

You might also like